Sei sulla pagina 1di 131

CBSE Sample Paper -01 (solved)

SUMMATIVE ASSESSMENT I
Class X Mathematics

Time allowed: 3 hours Maximum Marks: 90

General Instructions:

a) All questions are compulsory.


b) The question paper comprises of 31 questions divided into four sections A, B, C and D. You
are to attempt all the four sections.
c) Questions 1 to 4 in section A are one mark questions. These are MCQs. Choose the correct
option.
d) Questions 5 to 10 in section B are two marks questions.
e) Questions 11 to 20 in section C are three marks questions.
f) Questions 21 to 31 in section D are four marks questions.
g) There is no overall choice in the question paper. Use of calculators is not permitted.

SECTION A
1. If two zeros of the polynomial f(x) = x3 4x2 3x + 12 are 3 and 3 , then find its third
zero.
2. ABC is an isosceles triangle with AC = BC. If AB2 = 2AC2, prove that ABC is a right triangle.
3. Evaluate cos60cos30 + sin60sin30.
1
4. Prove that cot 2 = 1
sin2
5. Find the median of the daily wages of ten workers from the following data:
22, 25, 18, 20, 28, 15, 27, 10, 9, 16
SECTION B
6. The graph of y = ax2 + bx + c is given in the following figure. Identify the signs of a, b and c.

Material downloaded from http://myCBSEguide.com and http://onlineteachers.co.in


Portal for CBSE Notes, Test Papers, Sample Papers, Tips and Tricks
7. In the given figure, DE || BC and CD || EF. Prove that AD2 = AB AF.

8. If sin + sin2 = 1, find the value of cos12 + 3cos10 + 3cos8 + cos6 + 2cos4 + 2cos2 2
9. For the following grouped frequency distribution, find the mode.
Class 3-6 6-9 9-12 12-15 15-18 18-21 21-24
Frequency 2 5 10 23 21 12 3

10. ABC is a right triangle, right angled at C. If A = 30 and AB = 40 units, find the remaining two
sides and B of ABC.

SECTION C
11. Prove that 3 2 is irrational.
x y
12. Solve: + = 2; ax by = a2 b2
a b
13. The mean of the following frequency distribution is 1.46. Find the missing frequencies.
Number of accidents (x) 0 1 2 3 4 5 Total
Frequency (f) 46 f1 f2 25 10 5 200

14. A ladder 15 m long reaches a window which is 9 m above the ground on one side of a street.
Keeping its foot at same point, the ladder is turned to other side of the street to reach a
window 12 m high. Find the width of the street.

3
15. If sin(A + B) = 1 and cos (A B) = , 0 < A + B 90, A > B then find A and B.
2
16. Prove (sin + cosec)2 + (cos + sec)2 = 7 + tan2 + cot2
17. Find the values of x and y if the total frequency and the median of the following data is 100
and 525, respectively.

Material downloaded from http://myCBSEguide.com and http://onlineteachers.co.in


Portal for CBSE Notes, Test Papers, Sample Papers, Tips and Tricks
Class 0- 100- 200- 300- 400- 500- 600- 700- 800- 900-
interval 100 200 300 400 500 600 700 800 900 1000
Frequency 2 5 x 12 17 20 y 9 7 4

18. P and Q are points on sides AB and AC, respectively of ABC. If AP = 3 cm, PB = 6 cm, AQ = 5
cm and QC = 10 cm, show that BC = 3PQ.

19. If and are the zeros of the quadratic polynomial f(x) = 2x2 5x + 7, find the polynomial
whose zeros are 2 + 3 and 3 + 2.
20. Prove that 2(sin6 + cos6) 3(sin4 + cos4) + 1 = 0

SECTION D
21. Find all the zeros of the polynomial f(x) = 2x4 3x3 3x2 + 6x 2, if two of its zeros are 2

and 2 .
22. Show graphically that the system of equations 2x + 4y = 10; 3x + 6y = 12 has no solution.
23. Prove that if the corresponding sides of two triangles are proportional, then they are similar.
24. ABCD is a rhombus. Prove that AB2 + BC2 + CD2 + DA2 = AC2 + BD2

12
25. If cotB = , prove that tan2B sin2B = sin4Bsec2B.
5

Material downloaded from http://myCBSEguide.com and http://onlineteachers.co.in


Portal for CBSE Notes, Test Papers, Sample Papers, Tips and Tricks
26. If (secA + tanA)(secB + tanB)(secC + tanC) = (secA tanA)(secB tanB)(secC tanC), prove
that each of the side is equal to 1.
27. Apply step-deviation method to find the arithmetic mean of the following frequency
distribution.
Variate (x) 5 10 15 20 25 30 35 40 45 50
Frequency 20 43 75 67 72 45 39 9 8 6
(f)

2sin2 A + 3cot 2 A
28. If cosecA = 2 , find the value of .
4tan2 A cos2 A
29. Draw a cumulative frequency curve and cumulative frequency polygon for the following
frequency distribution by less than method.
Age (in years) 0-9 10-19 20-9 30-39 40-49 50-59 60-69
Number of persons 5 15 20 23 17 11 9

30. A train covered a certain distance at a uniform speed. If the train would have been 6 km/hr
faster, it would have taken 4 hours less than the scheduled time. And, if the train were slower
by 6 km/hr, it would have taken 6 hours more than the scheduled time. Find the length of the
journey.
31. The percentage of salary that 10 households donate to an orphanage is given below:
5, 3, 10, 5, 2, 4, 7, 8, 1, 5
Find the mean, median and mode of the data. Also tell the values depicted by the persons of
these households.

Material downloaded from http://myCBSEguide.com and http://onlineteachers.co.in


Portal for CBSE Notes, Test Papers, Sample Papers, Tips and Tricks
CBSE Sample Paper -01 (solved)
SUMMATIVE ASSESSMENT I
Class X Mathematics

Time allowed: 3 hours ANSWERS Maximum Marks: 90

SECTION A
1. Solution:
Let = 3 and = 3 be the given zeros and be the third zero. Then,

4 Coeff. of x 2
+ + = [Using + + = ]
1 Coeff. of x 3

3 3 +=4
=4
Hence, third zero is 4.
2. Solution:
We have AC = BC and AB2 = 2AC2
Now, AB2 = 2AC2
AB2 = AC2 + AC2
AB2 = AC2 + BC2 [ AC = BC (Given)]
ABC is a right triangle right angled at C.

3. Solution:
We have cos60cos30 + sin60sin30

1 3 3 1 3 3 3 3
= + = + = 2 =
2 2 2 2 4 4 4 2
4. Solution:
1
We have LHS = cot 2
sin2
1
= cot 2 cosec2 sin = cosec

= 1 = RHS 1 + cot 2 = cosec2 cot 2 cosec2= 1

5. Solution:

Material downloaded from http://myCBSEguide.com and http://onlineteachers.co.in


Portal for CBSE Notes, Test Papers, Sample Papers, Tips and Tricks
Arranging the wages in ascending order of magnitude, we have
9, 10, 15, 16, 18, 20, 22, 25, 27, 28
Since there are 10 observations, therefore, median is the arithmetic mean of
th th
10 10
and + 1 observations.
2 2
18 + 20
Thus, median = = 19
2
SECTION B
6. Solution:
We observe that y = ax2 + bx + c represents a parabola opening downwards. Therefore, a < 0.
We also observe that the vertex of the parabola is in first quadrant.
b
> 0 b < 0 b > 0
2a
Parabola y = ax2 + bx + c cuts Y-axis at P. On Y-axis, we have x = 0.
Putting x = 0 in y = ax2 + bx + c, we get y = c.
So, the coordinates of P are (0, c). As P lies on the positive direction of Y-axis, therefore, c > 0.
Hence, a < 0, b > 0 and c > 0.
7. Solution:
In ABC, we have DE || BC
AB AC
= [By basic proportionality theorem] (i)
AD AE
In ADC, we have FE || DC
AD AC
= [By basic proportionality theorem] (ii)
AF AE
From (i) and (ii), we get
AB AD
= AD2 = AB AF
AD AF
8. Solution:
We have sin + sin2 = 1 sin = 1 sin2 sin = cos2
Now, cos12 + 3cos10 + 3cos8 + cos6 + 2cos4 + 2cos2 2
= (cos12 + 3cos10 + 3cos8 + cos6) + 2(cos4 + cos2 1)
= (cos4 + cos2)3 + 2(cos4 + cos2 1)

Material downloaded from http://myCBSEguide.com and http://onlineteachers.co.in


Portal for CBSE Notes, Test Papers, Sample Papers, Tips and Tricks
= (sin2 + cos2)3 + 2(sin2 + cos2 1) [ cos2 = sin, cos4 = sin2]
= 1 + 2(1 1) = 1
9. Solution:
We observe that the class 12-15 has maximum frequency. Therefore, this is the modal class.
We have,
l = 12, h = 3, f = 23, f1 = 10 and f2 = 21
f f1
Mode = l + h
2 f f1 f2

23 10
= 12 + 3
46 10 21
13 13
= 12 + 3 = 12 + = 14.6
15 5
10. Solution:
In ABC, we have,
A + B + C = 180
30 + B + 90 = 180 [ A = 30 and C = 90]
B = 180 120 = 60
AC
Now, cosA =
AB
AC
cos30 =
40
3 AC
=
2 40
3
AC = 40 AC = 20 3 units
2
BC
And , sinA =
AB
BC
sin30 =
40
1 BC
=
2 40
1
BC = 40 BC = 20 units
2
Hence, AC = 20 3 units and BC = 20 units

Material downloaded from http://myCBSEguide.com and http://onlineteachers.co.in


Portal for CBSE Notes, Test Papers, Sample Papers, Tips and Tricks
SECTION C
11. Solution:
Let us assume, to the contrary, that 3 2 is rational. Then, there exist co-prime positive
integers a and b such that
a
3 2=
b
a
2=
3b
a
2 is rational [ 3, a and b are integers, is a rational number]
3b
This contradicts the fact that 2 is irrational. So, out assumption is not correct.
Hence, 3 2 is an irrational number.
12. Solution:
The given system of equations may be written as
bx + ay 2ab = 0
ax by (a2 b2) = 0
By cross multiplication, we have
x y 1
= =
a(a b ) ( b)( 2ab) b(a b ) a( 2ab) b( b) a(a)
2 2 2 2

x y 1
= = 2 2
a(a b ) 2ab b(a b ) + 2a b b a
22 2 2 2 2

x y 1
= =
a(a b + 2b ) b(a b 2a ) (a + b2 )
2 2 2 2 2 2 2

x y 1
= =
a(a + b )
2 2
b( a b ) (a + b2 )
2 2 2

a(a2 + b2 ) b(a2 + b2 )
x= = a and y = =b
(a2 + b2 ) (a2 + b2 )
Hence, solution of the given system of equations is x = a and y = b.

Material downloaded from http://myCBSEguide.com and http://onlineteachers.co.in


Portal for CBSE Notes, Test Papers, Sample Papers, Tips and Tricks
13. Solution:
Calculation of mean
xi fi fixi
0 46 0
1 f1 f1
2 f2 2f2
3 25 75
4 10 40
5 5 25
N = 86 + f1 + f2 fixi = 140 + f1 + 2f2
We have, N = 200
200 = 86 + f1 + f2
f1 + f2 = 114 (i)
Also, mean = 1.46

1.46 =
fx i i

N
140 + f1 + 2 f2
1.46 =
200
292 = 140 + f1 + 2f2
f1 + 2f2 = 152 (ii)
From (i), f1 = 114 f2
Putting the value of f1 in (ii), we have
114 f2 + 2f2 = 152
f2 = 152 114 = 38
Putting the value of f2 in (i), we have
f1 + 38 = 114
f1 = 114 38 = 76
Thus, we have f1 = 76 and f2 = 38.
14. Solution:
Let AB be the width of the street and C be the foot of the ladder. Let D and E be the windows
at heights of 9 m and 12 m, respectively from the ground. Then, CD and EF are the two
positions of the ladder.

Material downloaded from http://myCBSEguide.com and http://onlineteachers.co.in


Portal for CBSE Notes, Test Papers, Sample Papers, Tips and Tricks
Clearly, AD = 9 m, BE = 12 m, CD = CE = 15 m.

In ACD, we have
CD2 = AC2 + AD2
152 = AC2 + 92
AC2 =225 81 = 144
AC =12 m
In BCE, we have
CE2 = BC2 + BE2
152 = BC2 + 122
BC2 = 225 144 = 81
BC = 9 m
Hence, width of the street = AB = AC + CB = (12 + 9) m = 21 m
15. Solution:
We have sin(A + B) = 1
sin(A + B) = sin90
A + B = 90 (i)

3
And, cos(A B) =
2
cos(A B) = cos30
A B = 30 (ii)
Adding (i) and (ii), we get
(A + B) + (A B) = 90 + 30
2A = 120 A = 60
Putting A = 60 in (i), we get
60 + B = 90 B = 90 60 = 30
Thus, A = 60 and B = 30
16. Solution:
We have
LHS = (sin + cosec)2 + (cos + sec)2
= (sin2 + cosec2 + 2sincosec) + (cos2 + sec2 + 2cossec)

Material downloaded from http://myCBSEguide.com and http://onlineteachers.co.in


Portal for CBSE Notes, Test Papers, Sample Papers, Tips and Tricks
1 1
= sin2 + cosec2 + 2sin + cos + sec + 2cos
2 2

sin cos
= (sin2 + cosec2 + 2) + (cos2 + sec2 + 2)
= sin2 + cos2 + cosec2 + sec2 + 4
= 1 + (1 + cot2) + (1 + tan2) + 4 [ cosec2 = 1 + cot2, sec2 = 1 + tan2]
= 7 + cot2 + tan2
= RHS
17. Solution:
Calculation of median
Class intervals Frequency (f) Cumulative frequency (cf)
0-100 2 2
100-200 5 7
200-300 x 7+x
300-400 12 19 + x
400-500 17 36 + x
500-600 20 56 + x
600-700 y 56 + x + y
700-800 9 65 + x + y
800-900 7 72 + x + y
900-1000 4 76 + x + y
Total = 100
We have, N = fi = 100
76 + x + y = 100
x + y = 24
It is given that the median is 525. Clearly, it lies in the class 500-600.
l = 500, h = 100, f = 20, F = 36 + x and N = 100
N
F
Now, median = l + 2 h
f
100
(36 + x )
525 = 500 + 2 100
20

Material downloaded from http://myCBSEguide.com and http://onlineteachers.co.in


Portal for CBSE Notes, Test Papers, Sample Papers, Tips and Tricks
50 36 x
525 500 = 100
20
25 = (14 x) 5
25 = 70 5x
5x = 45
45
x= x=9
5
Putting x = 9 in x + y = 24, we get
9 + y = 24
y = 24 9 = 15
Thus, x = 9 and y = 15.
18. Solution:
We have,
AB = AP + PB = 3 + 6 = 9 cm
And, AC = AQ + QC = 5 + 10 = 15 cm
AP 3 1 AQ 5 1
= = and = =
AB 9 3 AC 15 3
AP AQ
=
AB QC
Thus, in triangles APQ and ABC, we have
AP AQ
= and A = A
AB QC
Therefore, by SAS criterion of similarity, we have
APQ ABC
AP PQ AQ
= =
AB BC AC
PQ AQ
=
BC AC
PQ 5 1
= =
BC 15 3
BC = 3PQ
19. Solution:
Since and are the zeros of the quadratic polynomial f(x) = 2x2 5x + 7

Material downloaded from http://myCBSEguide.com and http://onlineteachers.co.in


Portal for CBSE Notes, Test Papers, Sample Papers, Tips and Tricks
5 5 7
+ = = and =
2 2 2
Let S and P denote respectively the sum and product of zeros of the required polynomial.
5 25
Then, S = (2 + 3) + (3 + 2) = 5( + ) = 5 =
2 2
And, P = (2 + 3)(3 + 2)
= 6(2 + 2) + 13
= 62 + 62 + 12 +
= 6( + )2 +
2
5 7 25 7 75 7
= 6 + = 6 + = + = 41
2 2 4 2 2 2
Hence, the required polynomial is given by
g(x) = k(x2 Sx + P)
25
= x 2 x + 41 , where k is any non-zero real number.
2
20. Solution:
We have
LHS = 2(sin6 + cos6) 3(sin4 + cos4) + 1
= 2[(sin2)3 + (cos2)3] 3(sin4 + cos4) + 1
= 2[(sin2 + cos2){(sin2)2 + (cos2)2 sin2 + cos2}] 3(sin4 + cos4) + 1
= 2{(sin2)2 + (cos2)2 sin2cos2} 3(sin4 + cos4) + 1
= 2sin4 + 2cos4 2sin2cos2 3sin4 3cos4 + 1
= sin4 cos4 2sin2cos2 + 1
= (sin4 + cos4 + 2sin2cos2) + 1
= (sin2 + cos2)2 + 1
= 1 + 1 = 0 = RHS
SECTION D
21. Solution:
We know that, if x = is a zero of a polynomial, then x is a factor of f(x). Since 2 and
2 are zeros of f(x), therefore, (x 2 )(x + 2 ) = x2 2 is a factor of f(x).
Now, we divide f(x) = 2x4 3x3 3x2 + 6x 2 by g(x) = x2 2 to find the other zeros of f(x).

Material downloaded from http://myCBSEguide.com and http://onlineteachers.co.in


Portal for CBSE Notes, Test Papers, Sample Papers, Tips and Tricks
We have, 2 x 2 3x + 1
x 2 2 2x 4 3 x 3 3x 2 + 6 x 2
2x 4 4x 2
+
3x 3 + x 2 + 6 x 2
3x 3 + 6x
+
x2 2
x 2
2
+
0
By division algorithm, we have
2x4 3x3 3x2 + 6x 2 = (x2 2)(2x2 3x + 1)
2x4 3x3 3x2 + 6x 2 = (x 2 )(x + 2 )(2x2 2x x + 1)

2x4 3x3 3x2 + 6x 2 = (x 2 )(x + 2 ){2x(x 1) (x 1)}

2x4 3x3 3x2 + 6x 2 = (x 2 )(x + 2 )(x 1)(2x 1)


1
Hence, the zeros of the given polynomial are 2, 2, 1 and .
2
22. Solution:
Graph of 2x + 4y = 10:
We have,
10 2x 5 x
2x + 4y = 10 4y = 10 2x y= y=
4 2
5 x 51 4
When x = 1, we have y = = = =2
2 2 2
5 x 53 2
When x = 3, we have y = = = =1
2 2 2
Thus, we have the following table: x 1 3
y 2 1

Graph of 3x + 6y = 12:
We have,

Material downloaded from http://myCBSEguide.com and http://onlineteachers.co.in


Portal for CBSE Notes, Test Papers, Sample Papers, Tips and Tricks
12 3x 4x
3x + 6y = 12 6y = 12 3x y= y=
6 2
4 x 42 2
When x = 2, we have y = = = =1
2 2 2
4 x 40 4
When x = 0, we have y = = = =2
2 2 2
Thus, we have the following table: x 2 0
y 1 2

Plot the points A (1, 2) and B (3, 1) of 2x + 4y = 10 on a graph paper. Join A and B and extend
it on both sides as shown in the figure.
Also, plot the points C (2, 1) and D (0, 2) of 3x + 6y = 12 on the same graph paper. Join C and
D and extend it on both sides as shown in the figure.

We find that the lines represented by equations 2x + 4y = 10 and 3x + 6y = 12 are parallel. So,
the two lines have no common point. Hence, the given system of equations has no solution.
23. Solution:
AB BC AC
Given: Two triangles ABC and DEF such that = =
DE EF DF

Material downloaded from http://myCBSEguide.com and http://onlineteachers.co.in


Portal for CBSE Notes, Test Papers, Sample Papers, Tips and Tricks
To prove: ABC DEF
Construction: Let P and Q be points on DE and DF respectively such that DP = AB and DQ =
AC. Join PQ,
AB AC
Proof: We have =
DE DF
DP DQ
= [ AB = DP and AC = DQ]
DE DF
PQ || EF [By the converse of Thales theorem]
DPQ = E and DQP = F [Corresponding angles]
Thus, in triangles DPQ and DEF, we have DPQ = E and DQP = F.
Therefore, by AA-criterion of similarity, we have
DPQ DEF (i)
DP PQ
= [By definition of similarity]
DE EF
AB PQ
= [ DP = AB]
DE EF
AB BC
But, =
DE EF
PQ BC
=
EF EF
PQ = BC
Thus, in triangles ABC and DPQ, we have
AB = DP, AC = DQ and BC = PQ
Therefore, by SSS criterion of congruence, we have
ABC DPQ (ii)
From (i) and (ii), we have
ABC DPQ and DPQ DEF
ABC DPQ and DPQ DEF [ ABC DPQ ABC DPQ]
ABC DEF

Material downloaded from http://myCBSEguide.com and http://onlineteachers.co.in


Portal for CBSE Notes, Test Papers, Sample Papers, Tips and Tricks
24. Solution:
Let the diagonals AC and BD of rhombus ABCD intersect at O.
Since the diagonals of a rhombus bisect each other at right angles.
AOB = BOC = COD = DOA = 90 and AO = CO, BO = OD.
Since AOB is a right triangle right angled at O,
AB2 = OA2 + OB2
2 2
1 1
AB2 = AC + BD [ OA = OC and OB = OD]
2 2
4AB2 = AC2 + BD2 (i)
Similarly, we have
4BC2 = AC2 + BD2 (ii)
4CD2 = AC2 + BD2 (iii)
4AD2 = AC2 + BD2 (iv)
Adding (i), (ii), (iii) and (iv), we get
4(AB2 + BC2 + CD2 + AD2) = 4(AC2 + BD2)
AB2 + BC2 + CD2 + AD2 = AC2 + BD2

25. Solution:
We have,
Base 12
cotB = =
Perpendicular 5
So, we draw a right triangle ABC, right angled at C such that
Base = BC = 12 units and Perpendicular = AC = 5 units
By Pythagoras theorem, we have
AB2 = BC2 + AC2
= 122 + 52 = 169

AB = 169 = 13
AC 5 AC 5 AB 13
sinB = = , tanB = = and secB = =
AB 13 BC 12 BC 12
Now, LHS = tan2B sin2B
= (tanB)2 (sinB)2

Material downloaded from http://myCBSEguide.com and http://onlineteachers.co.in


Portal for CBSE Notes, Test Papers, Sample Papers, Tips and Tricks
2 2
5 5
=
12 13
25 25
=
144 169
1 1
= 25
144 169
169 144
= 25
144 169
25 25 25 52 52
= 25 = = 2 (i)
144 169 144 169 12 132
And, RHS = sin4Bsec2B
= (sinB)4(secB)2
4 2
5 13 54 132
= = 4
13 12 13 12
2

54 52 52
= 2 = (ii)
13 122 132 122
From (i) and (ii), we have tan2B sin2B = sin4Bsec2B.
26. Solution:
We have,
(secA + tanA)(secB + tanB)(secC + tanC) = (secA tanA)(secB tanB)(secC tanC)
Multiplying both sides by (secA tanA)(secB tanB)(secC tanC), we get
(secA + tanA)(secB + tanB)(secC + tanC) (secA tanA)(secB tanB)(secC tanC)
= (secA tanA)2(secB tanB)2(secC tanC)2
(sec2A tan2A)(sec2B tan2B)(sec2C tan2C)
= (secA tanA)2(secB tanB)2(secC tanC)2
1 = [(secA tanA)(secB tanB)(secC tanC)]2
(secA tanA)(secB tanB)(secC tanC) = 1
Similarly, multiplying both sides by (secA + tanA)(secB + tanB)(secC + tanC), we get
(secA + tanA)(secB + tanB)(secC + tanC) = 1

27. Solution:
Let the assumed mean be A = 25 and h = 5.

Material downloaded from http://myCBSEguide.com and http://onlineteachers.co.in


Portal for CBSE Notes, Test Papers, Sample Papers, Tips and Tricks
Calculation of mean
Variate Frequency Deviations x i 25 fiui
ui =
xi fi di = xi 25 5

5 20 20 4 80
10 43 15 3 129
15 75 10 2 150
20 67 5 1 67
25 72 0 0 0
30 45 5 1 45
35 39 10 2 78
40 9 15 3 27
45 8 20 4 32
50 6 25 5 30
N = fi = 384 fiui = 214
We have,
N = 384, A = 25, h = 5 and fiui = 214
1
Mean = X = A + h f i ui
N
214
= 25 + 5
384
= 25 2.786 = 22.214
28. Solution:
We have,
1 1
cosecA = 2 = 2 sinA =
sin A 2

Now, cosA = 1 sin2 A


2
1 1
= 1 =
2 2

1
sin A
tanA = = 2 =1
cos A 1
2

Material downloaded from http://myCBSEguide.com and http://onlineteachers.co.in


Portal for CBSE Notes, Test Papers, Sample Papers, Tips and Tricks
1 1
And, cotA = = =1
tan A 1
2
1
+ 3( 1 )
2
2
2sin2 A + 3cot 2 A 2
Hence, =
4tan2 A cos2 A 1
2

4 (1 )
2

2
1
2 + 3
2 1+3 4 8
= = = =
1 7 7 7
4
2 2 2
29. Solution:
The given frequency distribution is not continuous. So, we first make it continuous and
prepare the cumulative frequency distribution as under.
Age (in years) Frequency Age less than Cumulative frequency
0.5-9.5 5 9.5 5
9.5-19.5 15 19.5 20
19.5-29.5 20 29.5 40
29.5-39.5 23 39.5 63
39.5-49.5 17 49.5 80
49.5-59.5 11 59.5 91
59.5-69.5 9 69.5 100

Now, we plot points (9.5, 5), (19.5, 20),


(29.5, 40), (39.5, 63), (49.5, 80), (59.5,
91) and (69.5, 100) and join them by a
free hand smooth curve to obtain the
required ogive as shown in the figure.
The cumulative frequency polygon is
obtained by joining these points by
line segments as shown below.

Material downloaded from http://myCBSEguide.com and http://onlineteachers.co.in


Portal for CBSE Notes, Test Papers, Sample Papers, Tips and Tricks
30. Solution:
Let the actual speed of the train be x km/hr and the actual time taken by y hours. Then,
Distance covered = (xy) km (i) [ Distance = Speed Time]
If the speed is increased by 6 km/hr, then time of journey is reduced by 4 hours, i.e., when
speed is (x + 6) km/hr, time of journey is (y 4) hours.
Distance covered = (x + 6)(y 4)
xy = (x + 6)(y 4) [using (i)]
4x + 6y 24 = 0
2x + 3y 12 = 0 (ii)
When the speed is reduced by 6 km/hr, then the time of journey is increased by 6 hours, i.e.,
when speed is (x 6) km/hr, time of journey is (y + 6) hours.
Distance covered = (x 6)(y + 6)
xy = (x 6)(y + 6) [using (i)]
6x 6y 36 = 0
xy6=0 (iii)
Thus, we obtain the following system of equations:
2x + 3y 12 = 0
xy6=0
By using cross-multiplication, we have,
x y 1
= =
3 6 ( 1) 12 2 6 1 12 2 1 1 3

Material downloaded from http://myCBSEguide.com and http://onlineteachers.co.in


Portal for CBSE Notes, Test Papers, Sample Papers, Tips and Tricks
x y 1
= =
30 24 1
x = 30 and y = 24.
Putting the values of x and y in equation (i), we get
Distance = 30 24 = 720 km
Hence, the length of the journey is 720 km.
31. Solution:
5 + 3 + 10 + 5 + 2 + 4 + 7 + 8 + 1 + 5 50
Mean of the given data = = =5
10 10
Arranging the given data in ascending order, we get
1, 2, 3, 4, 5, 5, 5, 7, 8, 10
We observe that maximum occurring observation is 5. Thus, mode of the given data is 5.
th th
n n
observation + + 1 observation
=
2 2
Median
2
5th observation + 6th observation 5 + 5 10
= = = =5
2 2 2
The values depicted by the persons of these households are social service and caring.

Material downloaded from http://myCBSEguide.com and http://onlineteachers.co.in


Portal for CBSE Notes, Test Papers, Sample Papers, Tips and Tricks
CBSE Sample Paper -02 (solved)
SUMMATIVE ASSESSMENT I
Class X Mathematics

Time allowed: 3 hours Maximum Marks: 90

General Instructions:

a) All questions are compulsory.


b) The question paper comprises of 31 questions divided into four sections A, B, C and D. You
are to attempt all the four sections.
c) Questions 1 to 4 in section A are one mark questions. These are MCQs. Choose the correct
option.
d) Questions 5 to 10 in section B are two marks questions.
e) Questions 11 to 20 in section C are three marks questions.
f) Questions 21 to 31 in section D are four marks questions.
g) There is no overall choice in the question paper. Use of calculators is not permitted.

SECTION A
13
1. Without actually performing long division, state whether will have terminating or
3125
non-terminating repeating decimal expansion. Also find the number of decimal places after
which the decimal expansion terminates.
2. Identify the given graph corresponds to a linear polynomial or a quadratic polynomial.

3. For what value of k, will the following system of equations has a unique solution?
x + 2y = 5
3x + ky = 15
4. Evaluate tan5tan25tan30tan65tan85.
5. Express sin81 + tan81 in terms of trigonometric ratios of angles between 0 and 45.

Material downloaded from http://myCBSEguide.com and http://onlineteachers.co.in


Portal for CBSE Notes, Test Papers, Sample Papers, Tips and Tricks
SECTION B
6. Find the HCF of 96 and 404 by prime factorisation method. Hence, find their LCM.

If tan2 = 1 a2 , prove that sec + tan3 cosec = ( 2 a2 ) 2 .


3
7.

8. Sum of two numbers if 35 and their difference is 13. Find the numbers.
9. The number of students absent in a school was recorded every day for 147 days and the raw
data was presented in the form of the following frequency table.
No. of students 5 6 7 8 9 10 11 12 13 15 18 20
absent
No. of days 1 5 11 14 16 13 10 70 4 1 1 1
Obtain the median and describe what information it conveys.
10. A man goes 10 m due east and then 24 m due north. Find the distance from the starting point.
SECTION C
11. Show that there is no positive integer n for which n 1 + n + 1 is rational.

12. ABC is a right triangle right angled at C and AC = 3 BC. Prove that ABC = 60.
13. The taxi charges in a city comprise of a fixed charge together with the charge for the distance
covered. For a journey of 10 km, the charge paid is Rs 75 and for a journey of 15 km, the
charge paid is Rs 110. What will a person have to pay for travelling a distance of 25 km?
14. If asec + btan + c = 0 and psec + qtan + r = 0, prove that
(br qc)2 (pc ar)2 = (aq bp)2
15. Through the mid-point M of the side CD of a parallelogram ABCD, the line BM is drawn
intersecting AC in L and AD produced in E. Prove that EL = 2BL.

16. Find the zeros of the polynomial f(u) = 4u2 + 8u and verify the relationship between the zeros
and its coefficients.
a2 b2
17. If sin= 2 2 , find the values of other five trigonometric ratios.
a +b

Material downloaded from http://myCBSEguide.com and http://onlineteachers.co.in


Portal for CBSE Notes, Test Papers, Sample Papers, Tips and Tricks
18. The following table gives weekly wages of workers in a certain organization. The frequency
of class 49-52 is missing. It is known that the mean of the frequency distribution is 47.2. Find
the missing frequency.
Weekly wages (Rs) 40-43 43-46 46-49 49-52 52-55
Number of workers 31 58 60 ? 27

19. Solve: ax + by = c
bx + ay = 1 + c
20. Without using trigonometric tables, evaluate
2 2 5
cosec2 58 cot58 tan32 tan13 tan37 tan45 tan53 tan77
3 3 3
SECTION D
1 2
21. Let a, b, c and p be rational numbers such that p is not a perfect cube. If a + bp + cp = 0 , then
3 3

prove that a = b = c = 0.
1
22. In a ABC, right angled at C, if tanA = , find the value of sinAcosB + cosAsinB.
3

Area ( DEF )
23. In the given figure, DE || BC and AD : DB = 5 : 4. Find .
Area ( CFB )

Material downloaded from http://myCBSEguide.com and http://onlineteachers.co.in


Portal for CBSE Notes, Test Papers, Sample Papers, Tips and Tricks
24. Find the mean marks of students from the following cumulative frequency distribution:
Marks Number of students Marks Number of students
0 and above 80 60 and above 28
10 and above 77 70 and above 16
20 and above 72 80 and above 10
30 and above 65 90 and above 8
40 and above 55 100 and above 0
50 and above 43

25. If cosec sin = l and sec cos = m, prove that l 2m2 ( l 2 + m2 + 3) = 1 .

26. Find the values of a and b so that x4 + x3 + 8x2 + ax + b is divisible by x2 + 1.


27. Draw the graphs of the following equations on the same graph paper.
2x + y = 2; 2x + y = 6
Find the coordinates of the vertices of the trapezium formed by these lines. Also, find the
area of the trapezium formed.
28. Prove that if in two triangles, one pair of corresponding sides are proportional and the
included angles are equal, then the two triangles are similar.
29. In a ABC, right angled at C and A = B,
(i) Is cosA = cosB? (ii) Is tanA = tanB?
What about other trigonometric ratios for A and B. Will they be equal?

30. A sweet seller has 420 kaju burfis and 130 badam burfis. She wants to stack them in such a
way that each stack has the same number and they take up the least area of the tray. What is
the number of burfis that can be placed in each stack for this purpose?
31. Rohans mother decided to distribute 900 bananas among patients of a hospital on her
birthday. If the female patients are twice the male patients and the male patients are thrice
the child patients in the hospital, each patient will get only one apple.
(i) Find the number of child patients, male patients and female patients in the hospital.
(ii) Which values are depicted by Rohans father in the question?

Material downloaded from http://myCBSEguide.com and http://onlineteachers.co.in


Portal for CBSE Notes, Test Papers, Sample Papers, Tips and Tricks
CBSE Sample Paper -02 (solved)
SUMMATIVE ASSESSMENT I
Class X Mathematics

Time allowed: 3 hours Answers Maximum Marks: 90

SECTION A
1. Solution:
We have,
13 13
= 0 5
3125 2 5
13
This shows that the prime factorization of the denominator of is of the form 2m 5n.
3125
Hence, it has terminating decimal expansion which terminates after 5 places of decimals.
2. Solution:
We observe that the graph y = f(x) is a parabola opening upwards. Therefore, f(x) is a
quadratic polynomial in which coefficient of x2 is positive.
3. Solution:
The given system of equations will have a unique solution if
1 2
k6
3 k
4. Solution:
We have,
tan5tan25tan30tan65tan85
= (tan5tan85)(tan25tan65)tan30
tan85 = tan ( 90 5 ) = cot 5
= (tan5cot5)(tan25cot25)tan30
tan65 = tan ( 90 25 ) = cot 25
1 1
= 11 =
3 3
5. Solution:
We have,
Sin81 + tan81
= sin(90 9) + tan(90 9)
= cos9 + cot9 [ sin(90 ) = cos and tan(90 ) = cot)

Material downloaded from http://myCBSEguide.com and http://onlineteachers.co.in


Portal for CBSE Notes, Test Papers, Sample Papers, Tips and Tricks
SECTION B
6. Solution:
We have,
96 = 2 2 2 2 2 3 = 25 3
404 = 2 2 101 = 22 101
HCF = 22 = 2 2 = 4
Now, HCF LCM = Product of the numbers
4 LCM = 96 404
96 404
LCM = = 96 101 = 9696
4
7. Solution:
We have,
sec + tan3 cosec
sec + tan3 cosec
= sec (Multiplying and dividing by sec)
sec
cos
= sec 1 + tan3 .
sin

= sec{1 + tan3 cot }

= 1 + tan2 {1 + tan2 }

( )
3
= 1 + tan2 2

{
= 1 + ( 1 a2 ) } = (2 a )
3 3
2 2 2

8. Solution:
Let the two numbers be x and y. Then,
x + y = 35
x y = 13
Adding equations (i) and (ii), we get
2x = 48 x = 24
Subtracting equation (ii) from equation (i), we get
2y = 22 y = 11
Hence, the two numbers are 24 and 11.

Material downloaded from http://myCBSEguide.com and http://onlineteachers.co.in


Portal for CBSE Notes, Test Papers, Sample Papers, Tips and Tricks
9. Solution:
Calculation of median
xi 5 6 7 8 9 10 11 12 13 15 18 20

fi 1 5 11 14 16 13 10 70 4 1 1 1
cf 1 6 17 31 47 60 70 140 144 145 146 147
We have,
N 147
N = 147 = = 73.5
2 2
N
The cumulative frequency just greater than is 140 and the corresponding value of
2
variable x is 12. Thus, the median = 12. This means that for about half the number of days,
more than 12 students were absent.
10. Solution:
Let the initial position of the man be O and his final position be B. Since the man goes 10 m
due east and then 24 m due north. Therefore, AOB is a right angled triangle right-angled at
A such that OA = 10 m and AB = 24 m.
By Pythagoras theorem, we have
OB2 = OA2 + AB2
OB2 = 102 + 242 = 100 + 576 = 676

OB = 676 = 26 m
SECTION C
11. Solution:
a
If possible, let there be a positive integer n for which n 1 + n + 1 is rational equal to
b
(say), where a, b are positive integers. Then,
a
= n 1 + n +1 (i)
b
b 1
=
a n1 + n+1

n+1 n1
{ }{ }
=
n+1 + n1 n+1 n1

Material downloaded from http://myCBSEguide.com and http://onlineteachers.co.in


Portal for CBSE Notes, Test Papers, Sample Papers, Tips and Tricks
n +1 n 1 n +1 n 1
= =
( n + 1 ) ( n 1) 2

2b
= n +1 n 1 (ii)
a
Adding (i) and (ii) and subtracting (ii) from (i), we get
a 2b a 2b
2 n+1 = + and 2 n 1 =
b a b a
a2 + 2b2 a2 2b2
n+1 = and n 1 =
2ab 2ab
a , b are integers
n + 1 and n 1 are rationals 2
a + 2b and a 2b are rationals.
2 2 2

2ab 2ab

(n + 1) and (n 1) are perfect squares of positive integers.


This is not possible as any two perfect squares differ at least by 3. Thus, there is no positive
integer n for which n 1 + n + 1 is rational.
12. Solution:
Let D be the mid-point of AB. Join CD. Since ABC is a right triangle right angled at C, therefore
AB2 = AC2 + BC2

AB2 = ( 3BC ) 2 + BC2

AB2 = 3BC2 + BC2


AB2 = 4BC2
AB = 2BC (i)
1
But, BD = AB AB = 2BD (ii)
2
From (i) and (ii), we have BC = BD.
We know that the mid-point of the hypotenuse of a right triangle is equidistant from the
vertices.
CD = AD = BD CD = BD
Thus, in ABC, we have
BD = CD = BC
BCD is an equilateral triangle.
ABC = 60

Material downloaded from http://myCBSEguide.com and http://onlineteachers.co.in


Portal for CBSE Notes, Test Papers, Sample Papers, Tips and Tricks
13. Solution:
Let the fixed charges of taxi be Rs x per km and the running charges be Rs y km/hr.
According to the given condition, we have
x + 10y = 75 (i)
x + 15y = 110 (ii)
Subtracting equation (ii) from equation (i), we get
5y = 35 y=7
Putting y = 7 in equation (i), we get x = 5.
Total charges from travelling a distance of 25 km
= x + 25y
= 5 + 25 7 = Rs 180
14. Solution:
asec + btan + c = 0
psec + qtan + r = 0
Solving these two equations for sec and tan by the cross-multiplication, we get
sec tan 1
= =
br qc cp ar aq bp
br cq cp ar
sec = and tan =
aq bp aq bp

Now, sec2 tan2 = 1


2 2
br cq cp ar
=1
aq bp aq bp

( br cq ) ( cp ar ) = ( aq bp )
2 2 2

15. Solution:
In BMC and EMD, we have
MC = MD [ M is the mid-point of CD]
CMB = EMD [Vertically opposite angles]
And, MBC = MED [Alternate angles]
So, by AAS-criterion of congruence, we have
BMC EMD
BC = DE (i)

Material downloaded from http://myCBSEguide.com and http://onlineteachers.co.in


Portal for CBSE Notes, Test Papers, Sample Papers, Tips and Tricks
Also, AD = BC [ ABCD is a parallelogram] (ii)
AD + DE = BC + BC
AE = 2BC (iii)
Now, in AEL and CBL, we have
ALE = CLB [Vertically opposite angles]
EAL = BCL [Alternate angles]
So, by AA-criterion of similarity of triangles, we have
AEL CBL
EL AE
=
BL CB
EL 2BC
= [Using equation (iii)]
BL BC
EL
=2 EL = 2BL
BL
16. Solution:
We have,
f(u) = 4u2 + 8u
= 4u(u + 2)
The zeros of f(u) are given by
f(u) = 0
4u(u + 2) = 0
u = 0 or u + 2 = 0
u = 0 or u = 2
Hence, the zeros of f(u) are:
= 0 and = 2
Now, + = 0 +(2) = 2 and = 0 2 = 0
Coefficient of u 8
Also, 2
= = 2
Coefficient of u 4
Constant term 0
And, = =0
Coefficient of u2 2
Coefficient of u
Sum of the zeros =
Coefficient of u2

Material downloaded from http://myCBSEguide.com and http://onlineteachers.co.in


Portal for CBSE Notes, Test Papers, Sample Papers, Tips and Tricks
Constant term
And, Product of the zeros =
Coefficient of u2
17. Solution:
We have,
Perpendicular a2 b2
sin= = 2 2
Hypotenuse a +b

So, draw a right triangle right angled at B such that


Perpendicular = a2 b2 , Hypotenuse = a2 + b2 and BAC =
By Pythagoras theorem, we have
AC2 = AB2 + BC2

(a ) ( )
2 2
2
+ b2 = AB2 + a2 b2

( ) (a )
2 2
AB2 = a2 + b2 2
b2

= ( a4 + b4 + 2a2b2 ) ( a4 + b4 2a2b2 )

= 4a2b2 = ( 2ab )
2

AB = 2ab
When we consider the trigonometric ratios of BAC = , we have
Base = AB = 2ab, Perpendicular = BC = a2 b2 and Hypotenuse = AC = a2 + b2
Base 2ab
cos = = 2 2
Hypotenuse a + b
Perpendicular a2 b2
tan = =
Base 2ab
Hypotenuse a2 + b2
cosec = = 2 2
Perpendicular a b
Hypotenuse a2 + b2
sec = =
Base 2ab
Base 2ab
cot = = 2 2
Perpendicular a b
18. Solution:
Let the missing frequency be f, the assumed mean be A = 47 and h = 3.
Calculation of mean

Material downloaded from http://myCBSEguide.com and http://onlineteachers.co.in


Portal for CBSE Notes, Test Papers, Sample Papers, Tips and Tricks
Class Mid-values fi di = xi 47.5 x i 47.5 fiui
ui =
intervals (xi) 3

40-43 41.5 31 6 2 62
43-46 44.5 58 3 1 58
46-49 47.5 60 0 0 0
49-52 50.5 f 3 1 f
52-55 53.5 27 6 2 54
N = fi = 176 + f fu
i i = f 66

We have,

X = 47.2, A = 47.5 and h = 3


1
X = A + h f i ui
N
f 66
47.2 = 47.5 + 3
176 + f
f 66
0.3 = 3
176 + f
1 f 66
=
10 176 + f

176 f = 10f 660


11f = 484 f = 44
Hence, the missing frequency is 44.
19. Solution:
The given system of equations may be written as
ax + by c = 0
bx + ay (1 + c) = 0
By cross multiplication, we have
x y 1
= =
b ( 1 + c ) a c a ( 1 + c ) b c a a b b

x y 1
= = 2 2
b (1 + c ) + ac a (1 + c ) + bc a b

Material downloaded from http://myCBSEguide.com and http://onlineteachers.co.in


Portal for CBSE Notes, Test Papers, Sample Papers, Tips and Tricks
x y 1
= = 2 2
ac bc b ac bc + a a b
x y 1
= =
c ( a b ) b c ( a b ) + a ( a b )( a + b )

c ( a b) b c (a b) + a
x= and y =
( a b )( a + b ) ( a b )( a + b )
c b c a
x= and y = +
a + b ( a b )( a + b ) a + b ( a b )( a + b )

Hence, the solution of the given system of equation is


c b c a
x= 2 2 and y = + 2 2
a+b a b a+b a b
20. Solution:
2 2 5
cosec2 58 cot58 tan32 tan13 tan37 tan45 tan53 tan77
3 3 3
2 2 5
= cosec2 58 cot58 tan ( 90 58 ) tan13 tan37 tan45
3 3 3
tan ( 90 37 ) tan ( 90 13 )

2 2 5
= cosec2 58 cot 2 58 tan13 tan37 tan45 cot37 cot13
3 3 3

=
2
3
( 5
)
cosec2 58 cot 2 58 tan13 tan37 1
3
1

1
tan37 tan13
2 5
= 1
3 3
2 5 2 5 3
= = = = 1
3 3 3 3
SECTION D
21. Solution:
We have,
1 2
a + bp + cp = 0
3 3
(i)
1
3
Multiplying both sides by p , we get
1 2
ap 3 + bp 3 + cp = 0 (ii)

Material downloaded from http://myCBSEguide.com and http://onlineteachers.co.in


Portal for CBSE Notes, Test Papers, Sample Papers, Tips and Tricks
Multiplying (i) by b and (ii) by c and subtracting, we get
1 2
1 2
2
ab + b p + bcp acp + bcp + c p = 0
2 3 3 3 3


1 1
(b 2
)
ac p + ab c p = 0
3 2 3
[ p is irrational]

b2 ac = 0 and ab c 2 p = 0

b2 = ac and ab = c 2 p

b2 = ac and a2b2 = c 4 p2

a2 ( ac ) = c 4 p2 Putting b2 = ac in a2b2 = c 4 p2

a3c c 4 p2 = 0

(a 3
)
c 3 p2 c = 0

a3 c 3 p2 = 0 or c = 0

Now, a3 c 3 p2 = 0

a3
p = 3
2

c
1
1
a3 3
(p ) 2 3
= 3
c
1
2
13 a 3
3

=

p
c
1
2
13 a 3
3

=

p
c
2
13 a
p =
c
1
a
This is not possible as p3 is irrational and is rational.
c
a3 c 3 p2 0 and hence c = 0

Putting c = 0 in b2 ac = 0 , we get b = 0.

Material downloaded from http://myCBSEguide.com and http://onlineteachers.co.in


Portal for CBSE Notes, Test Papers, Sample Papers, Tips and Tricks
1 2
Putting b = 0 and c = 0 in a + bp 3 + cp 3 = 0 , we get a = 0.
Hence, a = b = c = 0.
22. Solution:
1
Let us draw a ABC, right angled at C in which tanA = .
3
1
Now, tanA =
3
BC 1 BC
= tan A = AC
AC 3

BC = x and AC = 3x (i)
By Pythagoras theorem, we have
AB2 = AC2 + BC2

( )
2
= 3x + x2

= 3x2 + x2 = 4x2
AB =2x
To find the trigonometric ratios of A, we have

Base = AC = 3x , Perpendicular = BC = x and Hypotenuse = AB = 2x

BC x 1 AC 3x 3
sinA = = = and cosA = = =
AB 2x 2 AB 2x 2
When we consider the trigonometric ratios of B, we have

Base = BC = x, Perpendicular = AC = 3x and Hypotenuse = AB = 2x

BC x 1 AC 3x 3
cosB = = = and sinB = = =
AB 2x 2 AB 2x 2

1 1 3 3 1 3
sinAcosB + cosAsinB = + = + =1
2 2 2 2 4 4

23. Solution:
In ABC, we have,
DE || BC
ADE = ABC and AED = ACB [Corresponding angles]

Material downloaded from http://myCBSEguide.com and http://onlineteachers.co.in


Portal for CBSE Notes, Test Papers, Sample Papers, Tips and Tricks
Thus, in triangles ADE and ABC, we have
A = A [Common]
ADE = ABC
And, AED = ACB
AED ABC [By AAA similarity]
AD DE
=
AB BC
We have,
AD 5
=
DB 4
DB 4
=
AD 5
DB 4
+1 = +1
AD 5
DB+AD 4 + 5
=
AD 5
AB 9 AD 5
= =
AD 5 AB 9
DE 5
=
BC 9
In DFE and CFB, we have
1 = 3 [Alternate interior angles]
2 = 4 [Vertically opposite angles]
Therefore, by AA similarity criterion, we have
DFE CFB

Area ( DEF )
2
DE2 5 25
= 2 = =
Area ( CFB ) BC 9 81

24. Solution:
Here, we have the cumulative frequency distribution. So, first we convert it into an ordinary
frequency distribution. We observe that there are 80 students getting marks greater than or
equal to 0 and 77 students have secured 10 and more marks. Therefore, the number of
students getting marks between 0 and 10 is 80 77 = 3.
Similarly, the number of students getting marks between 10 and 20 is 77 72 = 5 and so on.

Material downloaded from http://myCBSEguide.com and http://onlineteachers.co.in


Portal for CBSE Notes, Test Papers, Sample Papers, Tips and Tricks
Thus, we obtain the following frequency distribution:
Marks Number of students Marks Number of students
0-10 3 50-60 15
10-20 5 60-70 12
20-30 7 70-80 6
30-40 10 80-90 2
40-50 12 90-100 8
Now, we compute arithmetic mean by taking 55 as the assumed mean.
Computation of mean
Marks (xi) Mid-value Frequency (fi) x i 55 fiui
ui =
10
0-10 5 3 5 15
10-20 15 5 4 20
20-30 25 7 3 21
30-40 35 10 2 20
40-50 45 12 1 12
50-60 55 15 0 0
60-70 65 12 1 12
70-80 75 6 2 12
80-90 85 2 3 6
90-100 95 8 4 32
Total fi = 80 fiui = 26
We have,
N = fi = 80, fiui = 26, A = 55 and h = 10
1
X = A + h fi ui
N
26
= 55 + 10 = 55 3.25 = 51.75 marks
80
25. Solution:
We have,
LHS = l 2m2 ( l 2 + m2 + 3) = 1

Material downloaded from http://myCBSEguide.com and http://onlineteachers.co.in


Portal for CBSE Notes, Test Papers, Sample Papers, Tips and Tricks
= (cosec sin)2(sec cos)2{(cosec sin)2 + (sec cos)2 + 3}

1 1
2

2
1 2
1
2

= sin cos sin + cos + 3
sin cos sin cos
2
1 sin2 1 cos2
2
1 sin2 2 1 cos2 2
= + + 3
sin cos sin cos
2
cos2 sin2
2
cos2 2 sin2 2
= + + 3
sin cos
sin cos

cos4 sin 4 cos4 sin4


= + + 3
sin2 cos2 sin2 cos2

cos6 +sin6 +3cos2 sin2


= cos2 sin2
cos2 sin2

= cos6 +sin6 +3cos2 sin2

= {(cos ) +(sin ) }+3cos sin


2
3
2
3
2 2

= {( cos +sin ) 3cos sin ( cos + sin )} +3cos sin


3
2 2 2 2 2 2 2 2

a3 + b3 = ( a + b )3 3ab ( a + b )

= (1 3cos2 sin2 ) +3cos2 sin2 cos2 +sin2 =1

= 1 = RHS
26. Solution:
If x4 + x3 + 8x2 + ax + b is exactly divisible by x2 + 1, then the remainder should be zero.
On dividing, we get
x2 + x + 7
x2 + 1 x 4 + x 3 + 8 x 2 + ax + b
x4 + x2

x 3 + 7 x 2 + ax + b
x3 +x
+
7 x 2 + x ( a 1) + b
7x 2 +7

x a 1 and
Material downloaded from http://myCBSEguide.com ( )
+ bhttp://onlineteachers.co.in
7
Portal for CBSE Notes, Test Papers, Sample Papers, Tips and Tricks
Quotient = x2 + x + 7 and Remainder = x(a 1) + (b 7)
Now, remainder = 0
x(a 1) + (b 7) = 0
x(a 1) + (b 7) = 0x + 0
a 1 = 0 and b 7 = 0
a = 1 and b = 7
27. Solution:
Graph of the equation 2x + y = 2:
When y = 0, we have x = 1
When x = 0, we have y = 2
Thus, we obtain the following table giving coordinates of two points on the line represented
by the equation 2x + y = 2.

x 1 0
y 0 2

Graph of the equation 2x + y = 6:


When y = 0, we get x = 3
When x = 0, we get y = 6
Thus, we obtain the following table giving coordinates of two points on the line represented
by the equation 2x + y = 6.
x 3 0
y 0 6

Plotting points A(1, 0) and B (0, 2) on the graph


paper on a suitable scale and drawing a line passing
through them, we obtain the graph of the line
represented by the equation 2x + y = 2 as shown in
the graph.
Plotting points C(3, 0) and D(0, 6) on the same
graph paper and drawing a line passing through
them, we obtain the graph of the line represented

Material downloaded from http://myCBSEguide.com and http://onlineteachers.co.in


Portal for CBSE Notes, Test Papers, Sample Papers, Tips and Tricks
by the equation 2x + y = 6 as shown in the graph.
Clearly, lines AB and CD form trapezium ACDB.
Also, area of trapezium ACDB = Area of OCD Area of OAB
1 1
= (OC OD) (OA OB)
2 2
1 1
= (3 6) (1 2) = 8 sq.units
2 2
28. Solution:
AB AC
Given: Two triangles ABC and DEF such that A = D and = .
DE DF
To prove: ABC DEF
Construction: Mark points P and Q on DE and DF, respectively such that DP = AB and DQ =
AC. Join PQ.

Proof: In triangles ABC and DPQ, we have


AB = DP, A = D and AC = DQ
Therefore, by SAS criterion of congruence, we have
ABC DPQ (i)
AB AC
Now, =
DE DF
DP DQ
= [ AB = DP and AC = DQ]
DE DF
PQ || EF [By the converse of Thales theorem]
DPQ = E and DQP = F [Corresponding angles]
Thus, in triangles DPQ and DEF, we have
DPQ = E and DQP = F

Material downloaded from http://myCBSEguide.com and http://onlineteachers.co.in


Portal for CBSE Notes, Test Papers, Sample Papers, Tips and Tricks
Therefore, by AAA criterion of similarity, we hve
DPQ DEF (ii)
From (i) and (ii), we get
ABC DPQ and DPQ DEF
ABC DPQ and DPQ DEF
ABC DEF

29. Solution:
We have,
A = B
BC = AC [ Sides opposite to equal angles are equal]
Let BC = AC = x (say)
Using Pythagoras theorem in ACB, we have
AB2 = AC2 + BC2
= x2 + x2
AB = 2x
(i) We have,
AC x 1
cosA = = =
AB 2x 2
BC x 1
cosB = = =
AB 2x 2
cosA = cosB
(ii) We have,
BC x
tanA = = =1
AC x
AC x
tanB = = =1
BC x
tanA = tanB
BC x 1 AC x 1
Now, sinA = = = and sinB = = =
AB 2x 2 AB 2x 2
sinA = sinB

Material downloaded from http://myCBSEguide.com and http://onlineteachers.co.in


Portal for CBSE Notes, Test Papers, Sample Papers, Tips and Tricks
AC x BC x
cotA = = = 1 and cotB = = =1
BC x AC x
cotA = cotB

AB 2x AB 2x
secA = = = 2 and secB = = = 2
AC x BC x
secA = secB

AB 2x AB 2x
cosecA = = = 2 and cosecB = = = 2
BC x AC x
cosecA = cosecB
30. Solution:
The area of the tray that is used up in stacking the burfis will be least if the seet seller stacks
maximum number of burfis in each stack. Since each stack must have the same number of
burfis, therefore, the number of stacks will be least if the number of burfis in each stack is
equal to the HCF of 420 and 130.
In order to find the HCF of 420 and 130, let us apply Euclids division lemma to 420 and 130
to get


3
420 = 130 3 + 130 (i) 130 420

390
30

Let us now consider the divisor 130 and the remainder 30 and apply division lemma to get


4
130 = 30 4 + 10 (ii) 30 130

120
10

Considering now divisor 30 and the remainder 10 and apply division lemma, we get


3
30 = 3 10 + 0 (iii) 10 30

30
0

Material downloaded from http://myCBSEguide.com and http://onlineteachers.co.in


Portal for CBSE Notes, Test Papers, Sample Papers, Tips and Tricks
Since, the remainder at this stage is zero. Therefore, last divisor 10 is the HCF of 420 and 130.
Hence, the sweet seller can make stacks of 10 burfis of each kind to cover the least area of the
tray.
31. Solution:
(i) Let the number of child patients in the hospital be x.
Then, the number of male patients = 3x
And, the number of female patients = 2(3x) = 6x
According to the question,
6x + 3x + x = 900
10x = 900
900
x= =90
10
Thus, the number of child patients in the hospital is 90.
And, the number of male patients = 3 90 = 270
The number of female patients = 2 270 = 540
(ii) The values depicted by Rohans father in the question are charity and empathy.

Material downloaded from http://myCBSEguide.com and http://onlineteachers.co.in


Portal for CBSE Notes, Test Papers, Sample Papers, Tips and Tricks
CBSE Sample Paper -03 (solved)
SUMMATIVE ASSESSMENT I
Class X Mathematics

Time allowed: 3 hours Maximum Marks: 90

General Instructions:

a) All questions are compulsory.


b) The question paper comprises of 31 questions divided into four sections A, B, C and D. You
are to attempt all the four sections.
c) Questions 1 to 4 in section A are one mark questions. These are MCQs. Choose the correct
option.
d) Questions 5 to 10 in section B are two marks questions.
e) Questions 11 to 20 in section C are three marks questions.
f) Questions 21 to 31 in section D are four marks questions.
g) There is no overall choice in the question paper. Use of calculators is not permitted.

SECTION A

1. Prove that cos1cos2cos3cos180 = 0.


2. Determine whether the triangle having sides 6 cm, 8 cm and 10 cm is a right triangle or not.
3. If tanA = cotB, prove that A + B = 90.
4. Find the value of x, if the mode of the following data is 25.
15, 20, 25, 18, 14, 15, 25, 15, 18, 16, 20, 25, 20, x, 18
5. If and are the zeros of the quadratic polynomial f(x) = x2 px + q, then find the value of
1 1
+ .

SECTION B

6. Following table shows the weight of the bags of 12 students:


Weight (Kg) 67 70 72 73 75
Number of students 4 3 2 2 1
Find the mean weight.

Material downloaded from http://myCBSEguide.com and http://onlineteachers.co.in


Portal for CBSE Notes, Test Papers, Sample Papers, Tips and Tricks
7. E is a point on side AD produced of a parallelogram ABCD and BE intersects CD at F. Prove
that ABE CFB.

8. In a rectangle ABCD, AB = 20 cm, BAC = 60. Calculate side BC.

9. There is a circular path around a sports field. Prenu takes 18 minutes to drive 1 round of the
field, while Raj takes 12 minutes for the same. Suppose they both start at the same point and
at the same time, and go in the same direction. After how many minutes will they meet again
at the starting point?
10. Find a cubic polynomial with the sum, sum of the products of its zeros taken two at a time,
and product of its zeros as 2, 7 and 14, respectively.

SECTION C
11. Prove that 2 + 5 is irrational.
sec + tan 1 cos
12. Prove that =
tan sec + 1 1 sin

13. If acos bsin = c, prove that asin + bcos = a2 + b2 c 2 .


14. Points A and B are 70 km apart on a highway. A car starts from a and another car starts from
B at the same time. If they travel in same direction, they meet in 7 hours but if they travel in
opposite direction, they meet in one hour. What are their speeds?
15. If the zeros of the polynomial x3 3x2 + x + 1 are a b, a and a + b. Find a and b.

Material downloaded from http://myCBSEguide.com and http://onlineteachers.co.in


Portal for CBSE Notes, Test Papers, Sample Papers, Tips and Tricks
16. If the diagonals of a quadrilateral divide each other proportionally, the quadrilateral is a
trapezium.
17. If the areas of two similar triangles are equal, prove that they are congruent.
cos45
18. Evaluate .
sec30 + cosec30
19. The following table gives production yield per hectare of wheat of 100 farms of a village.
Production yield (kg/ha) 50-55 55-60 60-65 65-70 70-75 75-80
Number of farms 2 8 12 24 38 16
Change the distribution to a more than type distribution and draw its ogive.
20. A 2-digit number is such that the product of its digits is 14. If 45 is added to the number, the
digits interchange their places. Find the number.

SECTION D

21. Prove that every positive integer different from 1 can be expressed as a product of non-
negative power of 2 and an odd number.
22. On dividing x 3 3x 2 + x + 2 by a polynomial g(x), the quotient and remainder were x 2 and
2x + 4, respectively. Find g(x).
23. State and prove Pythagoras theorem.
24. Students of a class are made to stand in rows. If one student is extra in a row, there would be
2 rows less. If one student is less in a row, there would be 3 rows more. Find the number of
students in the class.
25. If B and Q are acute angles such that sinB = sinQ, then prove that B = Q.

26. Solve the following system of equations in x and y


( a b) x + ( a + b ) y = a2 2ab b2
( a + b )( x + y ) = a2 + b2

Material downloaded from http://myCBSEguide.com and http://onlineteachers.co.in


Portal for CBSE Notes, Test Papers, Sample Papers, Tips and Tricks
27. During the medical checkup of 35 studetns of a class, their weights were recorded as follows:
Weight Less Less Less Less Less Less Less Less
(kg) than than than than than than than than
38 40 42 44 46 48 50 52
Number of 0 3 5 9 14 28 32 35
students
Draw a less than type ogive for the given data. Hence, obtain the median weight from the
graph and verify the result by using the formula.

28. If cot =
7
, evaluate
(1 + sin)(1 sin) .
8 (1 + cos)(1 cos)
29. Prove that the line segments joining the mid-points of the sides of a triangle form four
triangles each of which is similar to the original triangle.
30. Let a, b, c and d be positive rationals such that a + b = c + d , then either a = c and b = d or b
and d are squares of rationals.
31. A man hires a taxi to cover a certain distance. The fare is Rs 50 for first kilometre and Rs 25
for subsequent kilometers. Taking total distance covered as x km and total fare as y:
a. Write a linear equation for this.
b. The man covers a distance of 10 km and gave Rs 300 to the driver. Driver said It is
not the correct amount and returned him the balance. Find the correct fare and the
amount paid back by the driver.
c. Which values are depicted by the driver in the question?

Material downloaded from http://myCBSEguide.com and http://onlineteachers.co.in


Portal for CBSE Notes, Test Papers, Sample Papers, Tips and Tricks
CBSE Sample Paper -03 (solved)
SUMMATIVE ASSESSMENT I
Class X Mathematics

Time allowed: 3 hours ANSWERS Maximum Marks: 90

SECTION A

1. Solution:
We have
LHS = cos1cos2cos3cos180
= cos1cos2cos3cos89cos90cos91cos180
= cos1cos2cos30 cos90cos91cos180 = 0 = RHS
2. Solution:
We have,
a = 6 cm, b = 8 cm and c = 10 cm
Here, the larger side is c = 10 cm
We have, a2 + b2 = 62 + 82 = 36 + 64 = 100 = 102 = c 2
So, the triangle with the given sides is a right triangle.
3. Solution:
We have
tanA = cotB
tanA = tan(90 B) A = 90 B A + B = 90
4. Solution:
The frequency table of the given data is as given below:
Value (xi) 14 15 16 18 20 25 x
Frequency (fi) 1 3 1 3 3 3 1
It is given that the mode of the given data is 25. So, it must have the maximum frequency.
That is possible only when x = 25.
Thus, x = 25.
5. Solution:
Since and are the zeros of the polynomial f(x) = x2 px + q,

Material downloaded from http://myCBSEguide.com and http://onlineteachers.co.in


Portal for CBSE Notes, Test Papers, Sample Papers, Tips and Tricks
p q
+ = = p and = 1 = q
1
1 1 + p
Thus, + = =
q
SECTION B
6. Solution:
Calculation of arithmetic mean
Weight (Kg) Frequency fixi
xi fi
67 4 268
70 3 210
72 2 144
73 2 146
75 1 75
N = fi = 12 fixi = 843

Mean = X =
fx i i
=
843
= 70.25
N 12
7. Solution:
In triangles ABE and CFB, we have
AEB = CBF [Alternate angles]
A = C [Opposite angles of a parallelogram]
Thus, by AA-criterion of similarity, we have
ABE CFB
8. Solution:
In ABC, we have
AB = 20, BAC = 60
BC
tanBAC =
AB
BC
tan60 =
20
BC
3= BC = 20 3 cm
20

Material downloaded from http://myCBSEguide.com and http://onlineteachers.co.in


Portal for CBSE Notes, Test Papers, Sample Papers, Tips and Tricks
9. Solution:
Required number of minutes is the LCM of 18 and 12.
We have,
18 = 2 32 and 12 = 22 3
LCM of 18 and 12 = 22 32 = 36
Thus, Prenu and Raj will meet again at the starting point after 36 minutes.
10. Solution:
If , and are the zeros of a cubic polynomial f(x), then
f(x) = k { x 3 ( + + ) x 2 + ( + + ) x } ,

where k is any non-zero real number.


Here, + + = 2, + + = 7 and = 14

( )
f(x) = k x 3 2x 2 7 x + 14 , where k is any non-zero real number.

SECTION C

11. Solution:
Let us assume on the contrary that 2 + 5 is a rational number. Then, there exist co-prime
positive integers a and b such that
a
2+ 5=
b
a
2= 5
b

( 5)
2
a 2
2 =
b
a2 2a
2 +2=5
b2 b
a2 2a
2
3 = 2
b b
a2 3b2
= 2
2ab

Material downloaded from http://myCBSEguide.com and http://onlineteachers.co.in


Portal for CBSE Notes, Test Papers, Sample Papers, Tips and Tricks
a2 3b2
2 is rational number a and b are integers, is rational.
2ab

This contradicts the fact that 2 is irrational. So, our assumption is wrong.
12. Solution:
sec + tan 1
LHS =
tan sec + 1

( sec + tan) ( sec2 tan2 )


= [ sec2 tan2 = 1 ]
tan sec + 1

( sec + tan){1 ( sec tan)}


=
tan sec + 1

=
( sec + tan)( tan sec+1)
tan sec + 1
= sec + tan
1 sin
= +
cos cos
1 + sin
=
cos
1 + sin 1 sin
=
cos 1 sin
1 sin2 cos2 cos
= = =
cos (1 sin) cos (1 sin) 1 sin

13. Solution:
We have,
(acos bsin)2 + (asin + bcos)2
= a2cos2 + b2sin2 + a2sin2 + b2cos2
= a2(cos2 + sin2) + b2(cos2 + sin2)
= a2 + b2 [ cos2 + sin2 = 1]
c2 + (asin + bcos)2 = a2 + b2
(asin + bcos)2 = a2 + b2 c2

asin + bcos = a2 + b2 c 2

Thus, asin + bcos = a2 + b2 c 2

Material downloaded from http://myCBSEguide.com and http://onlineteachers.co.in


Portal for CBSE Notes, Test Papers, Sample Papers, Tips and Tricks
14. Solution:
Let the speed of faster car at A = x km/hr and the speed of slower car at B = y km.hr.
Case 1: When they travel in same direction

A B 7y
70 km
7x

Distance covered by faster car in 7 hours = 7x km


Distance covered by slower car in 7 hours = 7y km
7x = 7y + 70
7(x y) = 70
x y = 10 (i)
Case 2: When they travel in opposite direction
A 70 km B

x y
Distance travelled by faster car in 1 hour = x km
Distance travelled by slower car in 1 hour = y km
x + y = 70 (ii)
Adding (i) and (ii), we get
2x = 80 x = 40
Substituting x = 40 in (i), we get
40 y = 10 y = 40 10 = 30
Speeds of cars would be 40 km/hr and 30 km/hr.
15. Solution:
x3 3x2 + x + 1 is a cubic polynomial.

Sum of its zeros =


(
coefficient of x 2 ) = ( 3 ) = 3
coefficient of x 3 1

ab+a+a+b=3
3a = 3
a=3

Material downloaded from http://myCBSEguide.com and http://onlineteachers.co.in


Portal for CBSE Notes, Test Papers, Sample Papers, Tips and Tricks
( constant term ) 1
Also, product of its zeros = 3
= = 1
coefficient of x 1
(a b) a (a + b) = 1
a(a2 b2) = 1
1(1 b2) = 1 [ a = 1]
1 b2 = 1
b2 = 2
b= 2

Thus, a = 1 and b = 2 .
16. Solution:
Given: A quadrilateral ABCD whose diagonals AC and BC intersect at O such that
AO BO
=
OC OD
To prove: ABCD is a trapezium.
Construction: Through O, draw OE || AB.

By Basic Proportionality Theorem, we have


AO BE
=
OC EC
AO BO
But, = [Given]
OC OD
BE BO
=
EC OD
BE BO
Now, in BCD, we have =
EC OD
By Basic Proportionality Theorem, we have
OE || DC

Material downloaded from http://myCBSEguide.com and http://onlineteachers.co.in


Portal for CBSE Notes, Test Papers, Sample Papers, Tips and Tricks
Now, OE || AB [By construction]
and, OE || DC
AB || DC
Thus, ABCD is a trapezium.
17. Solution:
Given that ABC and DEF are two similar triangles and their areas are equal.
ABC DEF
arABC AB2 BC2 AC2
= = =
arDEF DE2 EF2 DF2
[Ratios of the areas of similar triangles is equal to the ratio of the squares of corresponding
sides]

But, arABC = arDEF


arABC
=1
arDEF
AB2 BC2 AC2
= = =1
DE2 EF2 DF2
Or, AB2 = DE2, BC2 = EF2 and AC2 = DF2
AB = DE
BC = EF
and, AC = DF
Thus, ABC DEF [By SSS criterion]
18. Solution:
1
cos45 2
=
sec30 + cosec30 2 2
+
3 1

Material downloaded from http://myCBSEguide.com and http://onlineteachers.co.in


Portal for CBSE Notes, Test Papers, Sample Papers, Tips and Tricks
1
= 2
2+2 3
3

1 3

( )
=
2 2 1+ 3

3
( )
=
2 2 1+ 3

(1 3 )
3

2
2 2 (1 + 3 ) (1 3 )
=
2

6 (1 3 )
=
4 ( 1 3)

=
6 ( 3 1 )
8

3 2 6
=
8
19. Solution:
Production yield Number of Production yield more Cumulative
(kg/ha) farms than or equal to (kg.ha) frequency (cf)
50-55 2 50 100
55-60 8 55 98
60-65 12 60 90
65-70 24 65 78
70-75 38 70 54
75-80 16 75 16
To draw the required ogive, we plot the points corresponding to the ordered pairs given by
(lower limit, corresponding frequency), i.e., (50, 100), (55, 98), (60, 90), (65, 78), (70, 54)
and (75, 16) on a graph paper and join them by freehand curve.

Material downloaded from http://myCBSEguide.com and http://onlineteachers.co.in


Portal for CBSE Notes, Test Papers, Sample Papers, Tips and Tricks
20. Solution:
Let the tens and units digits of the required number be x and y, respectively. Then xy = 14.
Required number = (10x + y)
Number obtained on reversing its digits = (10y + x)
(10x + y) + 45 = (10y + x)
9(y x) = 45
y x = 45 (i)

( y + x) ( y x)
2 2
Now, =4xy

( y + x) = ( y x)
2
+ 4 xy

= 25 + 4 14 = 81
y+x=9 (ii)
On adding (i) and (ii), we get
2y = 14 y=7

Material downloaded from http://myCBSEguide.com and http://onlineteachers.co.in


Portal for CBSE Notes, Test Papers, Sample Papers, Tips and Tricks
Putting y = 7 in (ii), we get
7+x=9 x=97=2
x = 2 and y = 9
SECTION D
21. Solution:
Let n be a positive integer other than 1. By the fundamental theorem of Arithmetic, n can be
uniquely expressed as powers of primes in ascending order. So, let
n = p1a1 p2a2 ...pk ak be the unique factorisation of n into primes with p1 < p2 < p3 < ... < pk .

Clearly, either p1 = 2 and p2, p3, , pk are odd positive integers or each of p1, p2, , pk is an odd
positive integer.
Therefore, we have the following cases:
Case I: When p1 = 2 and p2, p3, , pk are odd positive integers.
In this case, we have
n = 2a1 p2a2 p3a3 ...pk ak

(
n = 2a1 p2a2 p3a3 ...pk ak )
n = 2a1 An odd positive integer
n = (A non-negative power of 2) (An odd positive integer)
Case II: When each of p1, p2, p3, , pk is an odd positive integer.
In this case, we have
n = p1a1 p2a2 p3a3 ...pk ak

(
n = 22 p1a1 p2a2 p3a3 ...pk ak )
n = (A non-negative power of 2) (An odd positive integer)
Thus, in either case, n is expressible as the product of a non-negative power of 2 and an odd
positive integer.
22. Solution:
Let p(x) = x 3 3x 2 + x + 2
q(x) = x 2
r(x) = 2x + 4
by division algorithm,
p(x) = g(x) q(x) + r(x)

Material downloaded from http://myCBSEguide.com and http://onlineteachers.co.in


Portal for CBSE Notes, Test Papers, Sample Papers, Tips and Tricks
g(x) q(x) = p(x) r(x)
g(x)(x 2) = x 3 3x 2 + x + 2 ( 2x + 4 )

= x 3 3x 2 + x + 2 + 2x 4
= x 3 3x 2 + 3x 2
g(x) is a factor of x 3 3x 2 + 3x 2 other than (x 2).
Dividing x 3 3x 2 + 3x 2 by (x 2), we obtain g(x) as follows:

x2 x + 1
x 2 x 3 3x 2 + 3x 2
x 3 2x 2
+
x 2 + 3x 2
x 2 + 2x
+
x 2
x 2
+
0
g(x) = x2 x + 1
23. Solution:
Theorem: In a right angled triangle, the square of the hypotenuse is equal to the sum of the
squares of the other two sides.

Given: A right-angled triangle ABC in which B = 90.


To prove: (Hypotenuse)2 = (Base)2 + (Perpendicular)2
i.e., AC2 = AB2 + BC2
Construction: From B, draw BD AC.
Proof: In triangles ADB and ABC, we have

Material downloaded from http://myCBSEguide.com and http://onlineteachers.co.in


Portal for CBSE Notes, Test Papers, Sample Papers, Tips and Tricks
ADB = ABC [Each equal to 90]
and, A = A [Common]
So, by AA-similarity criterion, we have
ADB ABC
AD AB
= [ In similar triangles corresponding sides are proportional]
AB AC
AB2 = AD AC (i)
In triangles BDC and ABC, we have
CDB = ABC [Each equal to 90]
and, C = C [Common]
So, by AA-similarity criterion, we have
BDC ABC
DC BC
= [ In similar triangles corresponding sides are proportional]
BC AC
BC2 = AC DC (ii)
Adding equations (i) and (ii), we get
AB2 + BC2 = AD AC + AC DC
AB2 + BC2 = AC(AD + DC)
AB2 + BC2 = AC AC
AB2 + BC2 = AC2
Or, AC2 = AB2 + BC2
24. Solution:
Let the number of students be x and the number of rows be y.
x
Then, number of students in each row =
y

x
When one student is extra in each row, there are 2 rows less, i.e., when each row has + 1
y
students, the number of rows is ( y 2) .

Total number of students = No. of rows No. of students in each row


x
x = + 1 ( y 2)
y

Material downloaded from http://myCBSEguide.com and http://onlineteachers.co.in


Portal for CBSE Notes, Test Papers, Sample Papers, Tips and Tricks
2x
x= x + y 2
y
2x
+ y 2 = 0 (i)
y
If one student is less in each row, then there are 3 rows more, i.e., when each row has
x
1 students, the number of rows is ( y + 3) .
y
Total number of students = No. of rows No. of students in each row
x
x = 1 ( y + 3)
y
3x
x= x+ y 3
y
3x
y 3 = 0 (ii)
y
x
Putting = u in (i) and (ii), we get
y
2u + y 2 = 0 (iii)
and, 3u y 3 = 0 (iv)
Adding (iii) and (iv), we get
u5=0 u=5
Putting u = 5 in (iii), we get y = 12
x x
Now, u = 5 =5 =5 x = 60
y 12
Thus, the number of students in the class is 60.
25. Solution:
Consider two right triangles ABC and PQR such that sinB = sinQ.
We have,
AC PR
sinB = and sinQ =
AB PQ
sinB = sinQ
AC PR
=
AB PQ

Material downloaded from http://myCBSEguide.com and http://onlineteachers.co.in


Portal for CBSE Notes, Test Papers, Sample Papers, Tips and Tricks
AC AB
= = k (say) (i)
PR PQ

AC = kPR and AB = kPQ (ii)


Using Pythagoras theorem in triangles ABC and PQR, we have
AB2 = AC2 + BC2 and PQ2 = PR2 + QR2

BC = AB2 AC2 and QR = PQ 2 PR 2

BC AB2 AC2
=
QR PQ 2 PR 2

k 2PQ 2 k 2PR 2
=
PQ 2 PR 2

k PQ 2 PR 2
= =k (iii)
PQ 2 PR 2

From (i) and (iii), we have


AC AB BC
= =
PR PQ QR
ACB PRQ
B = Q
26. Solution:
The given system of equations may be written as

( a b ) x + ( a + b ) y ( a2 2ab b2 ) = 0
( a + b ) x + ( a + b ) y ( a2 + b2 ) = 0
By cross-multiplication, we have

x y
=
( ) ( ) ( )
( a + b ) a + b ( a + b ) a 2ab b ( a b ) a + b ( a + b) a2 2ab b2
2 2 2 2 2 2
( )
1
=
( a b )( a + b ) ( a + b )
2

x y
=
( ) (
( a + b ) a + b + ( a + b ) a 2ab b
2 2 2 2
) ( ) (
( a b ) a + b + ( a + b ) a2 2ab b2
2 2
)

Material downloaded from http://myCBSEguide.com and http://onlineteachers.co.in


Portal for CBSE Notes, Test Papers, Sample Papers, Tips and Tricks
1
=
( a b )( a + b ) ( a + b )
2

x y
=
( a + b ){( a 2
) (
+ b + a 2ab b
2 2 2
)} ( ) (
( a + b ) a 2ab b2 ( a b ) a2 + b2
2
)
1
=
( a + b )( a b a b )
x y 1
= 3 2 =
( 2
)
( a + b ) 2ab 2b a a b 3ab b a ab + a b + b ( a + b ) 2b
2 3 3 2 2 3

x y 1
= =
2b ( a + b )
2
4ab2
2b ( a + b )

2b ( a + b )
2
4ab2 2ab
x= = a + b and y = =
2b ( a + b ) 2b ( a + b ) a + b

2ab
Hence, the solution of the given system of equations is x = a + b, y = .
a+b
27. Solution:
To draw the required ogive, we plot the points (38, 0), (40, 3), (44, 9), (46, 14), (48, 28), (50,
32) and (52, 35) and join them by a freehand curve.

Material downloaded from http://myCBSEguide.com and http://onlineteachers.co.in


Portal for CBSE Notes, Test Papers, Sample Papers, Tips and Tricks
n 35
To obtain the value of the median, we locate the point = = 17.5 on the y-axis. From this
2 2
point, we draw a line parallel to the x-axis, meeting the ogive at the point P. From P, we draw
a perpendicular PM on the x-axis. The x-coordinate of the point where this perpendicular
meets the x-axis, i.e., M gives the value of the median.
The required value of the median is 46.5 kg.
Verification:
Weight (kg) Number of students (fi) Cumulative frequency (cf)
38-40 3 3
40-42 2 5
42-44 4 9
44-46 5 14
46-48 14 28
48-50 4 32
50-52 3 35
n
Here, n = 35, = 17.5
2
Median class is 46-48
l = 46, f = 14, cf = 14, h = 2
n
cf
Median = l+ 2 h
f

17.5 14
= 46 + 2
14
3.5
= 46 + 2
14
= 46.5 kg
The value of the median in both the cases is same, i.e., 46.5 kg.
Hence verified.

Material downloaded from http://myCBSEguide.com and http://onlineteachers.co.in


Portal for CBSE Notes, Test Papers, Sample Papers, Tips and Tricks
28. Solution:
Draw a right triangle ABC in which ABC =

AB 7
Since, cot = =
AC 8
Let AB = 7 units and AC = 8 units

BC = AB2 + AC2 (By Pythagoras Theorem)

= 72 + 82

= 49 + 64

= 113 units
AC 8 AB 7
sin = = and cos = =
BC 113 BC 113

Now,
(1 + sin)(1 sin) = 1 sin2
(1 + cos)(1 cos) 1 cos2
2
8
1
= 113
2
7
1
113
64
1
= 113
49
1
113
113 64 49
= =
113 49 64
29. Solution:
Given: ABC in which D, E and F are the mid-points of sides BC, CA and AB, respectively.
To prove: Each of the triangles AFE, FBD, EDC and DEF is similar to ABC.
Proof: Consider triangles AFE and ABC.

Material downloaded from http://myCBSEguide.com and http://onlineteachers.co.in


Portal for CBSE Notes, Test Papers, Sample Papers, Tips and Tricks
Since F and E are the mid-points of AB and AC, respectively
FE || BC
AFE = B [Corresponding angles]
Thus, in AFE and ABC, we have
AFE = B
and, A = A [Common]
Similarly, we have
FBD ABC and EDC ABC.
Now, we shall show that DEF ABC.
Clearly, ED || AF and DF || EA.
AFDE is a parallelogram.
EDF = A [ Opposite angles of a parallelogram are equal.]
Similarly, BDEF is a parallelogram.
DEF = B [ Opposite angles of a parallelogram are equal.]
Thus, in triangles DEF and ABC, we have
EDF = A and DEF = B
So, by AA-criterion of similarity, we have
DEF ABC.
Thus, in each one of the triangles AFE, FBD, EDC and DEF is similar to ABC.
30. Solution:
If a = c, then
a+ b =c + d b= d b=d
So, let a c. Then, there exists a positive rational number x such that a = c + x.

Now, a + b = c + d

Material downloaded from http://myCBSEguide.com and http://onlineteachers.co.in


Portal for CBSE Notes, Test Papers, Sample Papers, Tips and Tricks
c+x + b =c+ d

x+ b= d (i)

(x + b) = ( d )
2 2

x 2 + 2 bx + b = d

d x 2 b = 2x b

d x2 b
b=
2x
d x2 b
b is rational. d , x and b are rationals, is rational
2x
From (i), we have
d =x+ b

d is rational
d is the square of a rational number.
Thus, either a = c and b = d or b and d are the squares of rationals.
31. Solution:
a. According to the given condition:
y = 50 + 25(x 1)
= 50 + 25x 25
y = 25x + 25
b. Correct fare = 25 10 + 25
= 250 + 25
= Rs 275
Amount paid back by the driver = 300 275 = Rs 25
c. The values depicted by the driver in the question are honesty and truthfulness.

Material downloaded from http://myCBSEguide.com and http://onlineteachers.co.in


Portal for CBSE Notes, Test Papers, Sample Papers, Tips and Tricks
CBSE Sample Paper -04 (solved)
SUMMATIVE ASSESSMENT I
Class X Mathematics

Time allowed: 3 hours Maximum Marks: 90

General Instructions:

a) All questions are compulsory.


b) The question paper comprises of 31 questions divided into four sections A, B, C and D. You
are to attempt all the four sections.
c) Questions 1 to 4 in section A are one mark questions. These are MCQs. Choose the correct
option.
d) Questions 5 to 10 in section B are two marks questions.
e) Questions 11 to 20 in section C are three marks questions.
f) Questions 21 to 31 in section D are four marks questions.
g) There is no overall choice in the question paper. Use of calculators is not permitted.

SECTION A

1. From the given graph, find the number of zeroes of the corresponding polynomial.

2. Prove that tan1tan2tan3tan89 = 1.


3. Express 0.6 as rational number in simplest form.
4. Express sec67+cosec58 in terms of trigonometric ratios of angles between 0 and 45.

Material downloaded from http://myCBSEguide.com and http://onlineteachers.co.in


Portal for CBSE Notes, Test Papers, Sample Papers, Tips and Tricks
5. In the given figure, AE is the bisector of the exterior CAD meeting BC produced in E. If AB =
10 cm, AC = 6 cm and BC = 12 cm, find CE.

SECTION B

6. If p(x) = 2x2 3x + 4, find p(3) and p(1).


7. The sum of two numbers is 100 and the difference between their squares is 256000. Find the
numbers.
8. Taking A = 60 and B = 30, verify that
sin(A B) = sinAcosB cosAsinB
9. The hypotenuse of a right triangle is 6 m more than the twice of the shortest side. If the third
side is 2 m less than the hypotenuse, find the sides of the triangle.
10. The following data gives the distribution of total household expenditure (in rupees) of
manual workers in a city:
Expenditure (Rs) Frequency Expenditure (Rs) Frequency
1000-1500 24 3000-3500 30
1500-2000 40 3500-4000 22
2000-2500 33 4000-4500 16
2500-3000 28 4500-5000 7
Find the average expenditure which is being done by the maximum number of manual
workers.

SECTION C
1
11. The sum of two numbers is 16 and the sum of their reciprocals is . Find the numbers.
3

Material downloaded from http://myCBSEguide.com and http://onlineteachers.co.in


Portal for CBSE Notes, Test Papers, Sample Papers, Tips and Tricks
12. An equilateral triangle is inscribed in a circle of radius 6 cm. Find its side.

a
13. If sin = , 0 < < 90, find the values of cos and tan.
a2 + b2
14. A point O in the interior of a rectangle ABCD is joined with each of the vertices A, B, C and D.
Prove that OB2 + OD2 = OC2 + OA2

15. If 3 times the larger of the two numbers is divided by the smaller one, we get 4 as quotient
and 3 as the remainder. Also, if 7 times the smaller number is divided by the larger one, we
get 5 as quotient and 1 as the remainder. Find the numbers.
16. Draw the graph of the polynomial f(x) = 4x2 + 4x 1. Also, find the vertex of this parabola.
17. Prove that one of every three consecutive positive integers is divisible by 3.

18. If
cos
cos
= m and
cos
sin
( )
= n, show that m2 + n2 cos2 = n2 .

19. If the median of the following frequency distribution is 46, find the missing frequencies.
Variable:
Less Less Less Less Less Less Less Less Less
than20 than20 than20 than20 than20 than20 than20 than20 than20
0 4 16 30 46 66 82 92 100

Material downloaded from http://myCBSEguide.com and http://onlineteachers.co.in


Portal for CBSE Notes, Test Papers, Sample Papers, Tips and Tricks
20. Prove that if three or more parallel lines are intersected by two transversals, prove that the
intercepts made by them on the transversals are proportional.

SECTION D

21. If p is a prime number, then prove that p is irrational.

22. The mean of the following frequency table is 50. But the frequencies f1 and f2 in class 20-40
and 60-80 are missing. Find the missing frequencies.
Class 0-20 20-40 40-60 60-80 80-100 Total
Frequency 17 f1 32 f2 19 120
23. A boat covers 32 km upstream and 36 km downstream in 7 hours. Also, it covers 40 km
upstream and 48 downstream in 9 hours. Find the speed of the boat in still water and that of
the stream.
24. Prove that the areas of two similar triangles are in the ratio of the squares of the
corresponding angle bisector segments.
25. Rama went to a stationary stall and purchased 2 pencils and 3 erasers for Rs 9. Her friend
Sonal saw the new variety of pencils and erasers with Rama and she also bought 4 pencils
and 6 erasers of the same kind for Rs 18. Represent this situation algebraically and
graphically.
26. Prove that in any triangle, the sum of the squares of any two sides is equal to twice the
square of half of the third side together with twice the square of the median which bisects the
third side.
27. In the given figure, AD = DB and B is a right angle. Find sin2 + cos2.

Material downloaded from http://myCBSEguide.com and http://onlineteachers.co.in


Portal for CBSE Notes, Test Papers, Sample Papers, Tips and Tricks
cos A sin A
28. Prove + = cos A + sin A .
1 tan A 1 cot A
29. Compute the median from the following data:
Mid-value 115 125 135 145 155 165 175 185 195
Frequency 6 25 48 72 116 60 38 22 3
1
30. Verify that 3, 1 and are the zeros of the cubic polynomial p( x ) = 3x 3 5x 2 11x 3 and
3
then verify the relationship between the zeros and its coefficients.
31. a. After every 6 months, price of petrol increases at the rate of Rs 4 per litre. Taking price of
petrol in December 2010 as x and present price of petrol as y, form a linear equation showing
the price of petrol in December 2014.
b. Due to continuous rise in the price of petrol, people are more interesting in CNG whose
price is increasing at the rate of Rs 3 per litre in a year. Form a linear equation taking price of
CNG in December 2010 as a and in December 2014 as b.
c. Which value is depicted by using CNG over petrol?

Material downloaded from http://myCBSEguide.com and http://onlineteachers.co.in


Portal for CBSE Notes, Test Papers, Sample Papers, Tips and Tricks
CBSE Sample Paper -04 (solved)
SUMMATIVE ASSESSMENT I
Class X Mathematics

Time allowed: 3 hours ANSWERS Maximum Marks: 90

SECTION A

1. Solution:
Since the given graph intersects the x-axis at 3 points, the polynomial has 3 zeros.
2. Solution:
We have
LHS = tan1tan2tan3tan89
= tan1tan2tan3tan44tan45tan46tan88tan89
= (tan1.tan89)(tan2.tan88) (tan44tan46)tan45
= {tan1.tan(90 1)}{(tan2.tan(90 2)} {(tan44tan(90 44)}tan45
= {tan1.cot1)(tan2.cot2) (tan44cot44)tan45 [ tan(90 ) = cot]
= 1 = RHS [ tancot = 1 and tan45 = 1]
3. Solution:
Let x = 0.6 .
Then, x = 0.666 (i)
10x = 6.666 (ii)
On subtracting (i) from (ii), we get
6 2
9x = 6 x= =
9 3
2
Thus, 0.6 =
3
4. Solution:
sec67+cosec58= sec(90 23) + cosec(90 32)
= cosec23 + sec32
5. Solution:
Since AE is the bisector of the exterior CAD.
BE AB 12 + x 10
= = x = 18
CE AC x 6

Material downloaded from http://myCBSEguide.com and http://onlineteachers.co.in


Portal for CBSE Notes, Test Papers, Sample Papers, Tips and Tricks
SECTION B

6. Solution:
We have,
p(x) = 2x2 3x + 4
p(3) = 2 32 3 3 + 4
=299+4
= 18 9 + 4 = 22 9 = 13
p(1) = 2 (1)2 3 (1) + 4
=2+3+4
=9
7. Solution:
Let the larger number be x and the smaller number be y.
Then, x + y = 1000 (i)
And, x2 y2 = 256000 (ii)
On dividing (ii) by (i), we get
x 2 y 2 256000
=
x+ y 1000
x y = 256 (iii)
Adding (i) and (iii), we get
2x = 1256
x = 628
Substituting x = 628 in (i), we get y = 372
Thus, the required numbers are 628 and 372.
8. Solution:
A = 60 and B = 30
A B = 60 30 = 30
1
sin(A B) = sin30 =
2
sinAcosB cosAsinB = sin60cos30 cos60sin30

3 3 1 1
=
2 2 2 2

Material downloaded from http://myCBSEguide.com and http://onlineteachers.co.in


Portal for CBSE Notes, Test Papers, Sample Papers, Tips and Tricks
3 1 2 1
= = =
4 4 4 2
sin(A B) = sinAcosB cosAsinB
9. Solution:
Let the shortest side be x metres in length. Then,
Hypotenuse = (2x + 6) m and third side = (2x + 4) m
By Pythagoras theorem, we have

( 2x + 6 ) = x 2 + ( 2x + 4 )
2 2

4x 2 + 24x + 36 = x 2 + 4x 2 + 16x + 16
x 2 8x 20 = 0
( x 10)( x + 2) = 0
x = 10 or x = 2
x = 10
Thus, the sides of the triangles are 10 m, 26 m and 24 m.
10. Solution:
We observe that the class 1500-2000 has the maximum frequency 40. So, it is the modal class
such that
l = 1500, h = 500, f = 40, f1 = 24 and f2 = 33
ff1 40 24
Mode = l + h = 1500 + 500
2 f f1 f2 80 24 33

16
= 1500 + 500 = 1847.826
23

SECTION C
11. Solution:
Let the required numbers be x and y.
Then, x + y = 16
1 1 1 x+ y 1 16 1
And, + = = = xy = 48
x y 3 xy 3 xy 3
We can write

( x + y)
2
xy= 4xy

Material downloaded from http://myCBSEguide.com and http://onlineteachers.co.in


Portal for CBSE Notes, Test Papers, Sample Papers, Tips and Tricks
(16)
2
= 4 48

= 256 192 = 64 = 8
x + y = 16 (i)
xy=8 (ii)
Or, x + y = 16 (iii)
x y = 8 (iv)
On solving (i) and (ii), we get x = 12 and y = 4
On solving (iii) and (iv), we get x = 4 and y = 12
Thus, the required numbers are 12 and 4.
12. Solution:
Let ABC be an equilateral triangle inscribed in a circle of radius 6 cm. Let O be the centre of
the circle. Then,
OA = OB = OC = 6 cm
Let OD be perpendicular from O on side BC. Then, D is the mid-point of BC and OB and OC are
bisectors of B and C respectively.
OBD = 30
In OBD, right angled at D, we have
OBD = 30 and OB = 6 cm
BD
cosOBD =
OB
BD
cos30 =
6
BD = 6cos30

3
BD = 6 =3 3
2

BC = 2BD = 2 3 3 = 6 3

Thus, the side of the equilateral triangle is 6 3 .


13. Solution:
We have,
a
sin =
a2 + b2

Material downloaded from http://myCBSEguide.com and http://onlineteachers.co.in


Portal for CBSE Notes, Test Papers, Sample Papers, Tips and Tricks
cos = 1 sin2

a2
= 1
a2 + b2

b2 b
= =
a +b
2 2
a2 + b2
a

= a +b =
sin 2 2a
tan =
cos b b
a +b
2 2

14. Solution:
Let ABCD be the given rectangle and let O be a point within it. Join OA, OB, OC and OD.
Through O, draw EOF || AB. Then, ABFE is a rectangle.
In right triangles OEA and OFC, we have
OA2 = OE2 + AE2 amd OC2 = OF2 + CF2
OA2 + OC2 = (OE2 + AE2) + (OF2 + CF2)
OA2 + OC2 = OE2 + OF2 + AE2 + CF2 (i)
Now, in right triangles OFB and ODE, we have
OB2 = OF2 + FB2 and OD2 = OE2 + DE2
OB2 + OD2 = (OF2 + FB2) + (OE2 + DE2)
OB2 + OD2 = OE2 + OF2 + DE2 + BF2
= OE2 + OF2 + CF2 + AE2 [ DE = CF and AE = BF] (ii)
From (i) and (ii), we get
OA2 + OC2 = OB2 + OD2
15. Solution:
Let the larger number be x and smaller one be y. We know that
Dividend = (Divisor Quotient) + Remainder (i)
When 3x is divided by y, we get 4 as quotient and 3 as remainder. Therefore, by using (i), we
get
3x = 4y + 3 3x 4y 3 = 0 (ii)
When 7y is divided by x, we get 5 as quotient and 1 as remainder. Therefore, by using (i), we
get
7y = 5x + 1 5x 7y + 1 = 0 (iii)

Material downloaded from http://myCBSEguide.com and http://onlineteachers.co.in


Portal for CBSE Notes, Test Papers, Sample Papers, Tips and Tricks
Solving equations (ii) and (iii), by cross-multiplication, we get
x y 1
= =
4 21 3 + 15 21 + 20
x = 25 and y = 18
Thus, the required numbers are 25 and 18.
16. Solution:
Let y = f(x) or y = 4x2 + 4x 1
The following table gives the values of y for various values of x.
x 2 3 1 1 0 1 1 3 2 5 3

2 2 2 2 2
y = 4x2 + 4x 1 25 16 9 4 1 0 1 4 9 16 25
Thus, the following points lie on the graph of y = 4x2 + 4x 1:
3 1 1 3 5
(2, 25), , 16 , (1, 9), , 4 , (0, 1), ,0 , (1, 1), , 4 , (2, 9), , 16
2 2 2 2 2
and (3, 25).
Plot these points on a graph paper and draw a free hand smooth curve passing through these
points.

The shape of the curve is shown in the figure. It is a parabola opening downward having its
1
vertex at ,0 .
2

Material downloaded from http://myCBSEguide.com and http://onlineteachers.co.in


Portal for CBSE Notes, Test Papers, Sample Papers, Tips and Tricks
17. Solution:
Let n, n + 1 and n + 2 be three consecutive positive integers.
We know that n is of the form 3q, 3q + 1 or 3q + 2.
So, we have the following cases:
Case I: When n = 3q
In this case, n is divisible by 3 but n + 1 and n + 2 are not divisible by 3.
Case II: When n = 3q+ 1
In this case, n + 2 = 3q + 1 + 2 = 3(q + 1),
which is divisible by 3 but n and n + 1 are not divisible by 3.
Case III: When n = 3q + 2
In this case, n + 1 = 3q + 1 + 2 = 3(q + 1),
which is divisible by 3 but n and n + 2 are not divisible by 3.
Thus, one of n, n + 1 and n + 2 is divisible by 3.
18. Solution:
We have,
LHS = ( m2 + n2 ) cos2

cos2 cos2 2 cos cos


= 2
+ 2 cos m = cos and n = sin
cos sin

cos2 sin2 + cos2 cos2 2


= cos
cos2 sin2

sin2 + cos2 2
= cos2 2 2 cos
cos sin

1 2
= cos2 2 2
cos
cos sin
2
cos2 cos
= = = n = RHS
2

sin2 sin

19. Solution:
We are given the cumulative frequency distribution. So, we first construct a frequency table
from the given cumulative frequency distribution and then we will make necessary
computations to compute median.

Material downloaded from http://myCBSEguide.com and http://onlineteachers.co.in


Portal for CBSE Notes, Test Papers, Sample Papers, Tips and Tricks
Class intervals Frequency (f) Cumulative frequency (cf)
20-30 4 4
30-40 12 16
40-50 14 30
50-60 16 46
60-70 20 66
70-80 16 82
80-90 10 92
90-100 8 100
N = fi = 100
N
Here, N = fi = 100 = 50
2
N
We observe that the cumulative frequency just greater than = 50 is 66 and the
2
corresponding class is 60-70.
So, 60-70 is the median class.
l = 60, f = 20, F = 46 and h = 10
N
F
Now, median = l + 2 h
f
50 46
= 60 + 10 = 62
20
20. Solution:
Given: Three parallel lines l, m and n which are cut by the transversals AB and CD in P, Q, R
and E, F, G, respectively.
PQ EF
To prove: =
QR FG

Construction: Draw PL || CD meeting the lines m and n


in M and L, respectively.
Proof: Since PE || MF and PM || EF,
PMFE is a parallelogram.
PM = EF (i)

Material downloaded from http://myCBSEguide.com and http://onlineteachers.co.in


Portal for CBSE Notes, Test Papers, Sample Papers, Tips and Tricks
Also, MF || LG and ML || FG,
MLFG is a parallelogram.
ML = FG (ii)
Now, in PRL, we have
QM || RL
PQ PM
= [By Thales Theorem]
QR ML
PQ EF
= [Using (i) and (ii)]
QR FG
SECTION D

21. Solution:
Let p be a prime number and if possible, let p be rational.

m
Let its simplest form be p= , where m and n are integers having no common factor other
n
than 1, and n 0.
m
Then p=
n
m2
p= [On squaring both sides]
n2
pn2 = m2 (i)

p divides m2 [ p divides pn2]


p divides m [ p is prime and p divides m2 p divides m]
Let m = pq for some integer q.
Putting m = pq in (i), we get
pn2 = p2q2
n2 = pq2
p divides n2 [ p divides pq2]
p divides n [ p is prime and p divides n2 p divides n]
Thus, p is a common factor of m and n.
But this contradicts the fact that m and n have no common factor other than 1.

Material downloaded from http://myCBSEguide.com and http://onlineteachers.co.in


Portal for CBSE Notes, Test Papers, Sample Papers, Tips and Tricks
The contradiction arises by assuming that p is rational.

Thus, p is irrational.

22. Solution:
Let the assumed mean be A = 50 and h = 20.
Calculation of mean
Class Frequency Mid-values xi A fiui
ui =
fi h

0-20 17 10 2 34
20-40 f1 30 1 f1
40-60 32 50 0 0
60-80 f2 70 1 f2
80-100 19 90 2 38
N = fi = 68 + f1 + f2 fiui = 4 f1 + f2
We have,
N = fi = 120 [Given]
68 + f1 + f2 = 120
f1 + f2 = 52 (i)
Now,
Mean = 50
1
A + h f i ui = 50
N

4 f1 + f 2
50 + 20 = 50
120
4 f1 + f 2
50 + = 50
6
4 f1 + f 2
=0
6
4 f1 + f 2 = 0

f1 f 2 = 4 (ii)

Solving equations (i) and (ii), we get


f1 = 28 and f2 = 24

Material downloaded from http://myCBSEguide.com and http://onlineteachers.co.in


Portal for CBSE Notes, Test Papers, Sample Papers, Tips and Tricks
23. Solution:
Let the speed of the boat in still water be x km/hr and the speed of the stream be y km/hr.
Then,
Speed upstream = (x y) km/hr
Speed downstream = (x + y) km/hr
Now,
32
Time taken to cover 32 km upstream = hrs
x y
36
Time taken to cover 36 km downstream = hrs
x+ y
But, total time of journey is 7 hours
32 36
+ =7 (i)
x y x+ y
40
Time taken to cover 40 km upstream =
x y
48
Time taken to cover 48 km downstream =
x+ y
In this case, total time of journey is given to be 9 hours.
40 48
+ =9 (ii)
x y x+ y
1 1
Putting = u and = v in equations (i) and (ii), we get
x y x+ y
32u + 36v = 7 32u + 36v 7 = 0 (iii)
40u + 48v = 9 40u + 48v 9 = 0 (iv)
Solving these equations by cross-multiplication, we get
u v 1
= =
36 9 48 7 32 9 40 7 32 48 40 36
u v 1
= =
324 + 336 288 + 280 1536 1440
u v 1
= =
12 8 96
12 1 8 1
u= = and v = =
96 8 96 8

Material downloaded from http://myCBSEguide.com and http://onlineteachers.co.in


Portal for CBSE Notes, Test Papers, Sample Papers, Tips and Tricks
1 1 1
Now, u = = xy=8 (v)
8 x y 8
1 1 1
and, v= = x + y = 12 (vi)
12 x + y 12
Solving equations (v) and (vi), we get x = 10 and y = 2
Thus, speed of the boat in still water = 10 km/hr
Speed of the stream = 2 km/hr
24. Solution:
Given: ABC DEF and AX and DY are bisector of A and D respectively.
Area ( ABC ) AX 2
To prove: =
Area ( DEF ) DY2

Proof: Since the ratio of the areas of two similar triangles are equal to the ratio of the squares
of any two corresponding sides.
Area ( ABC ) AB2
= (i)
Area ( DEF ) DE2

Now, ABC DEF


A = D
1 1
A = D
2 2
BAX = EDY

Thus, in triangles ABX and DEY, we have


BAX = EDY and B = E [ ABC DEF]
So, by AA-similarity criterion, we have
ABX DEY
AB AX
=
DE DY

Material downloaded from http://myCBSEguide.com and http://onlineteachers.co.in


Portal for CBSE Notes, Test Papers, Sample Papers, Tips and Tricks
AB2 AX 2
= (ii)
DE2 DY2
From (i) and (ii), we get
Area ( ABC ) AX 2
=
Area ( DEF ) DY2

25. Solution:
Let the cost of 1 pencil be Rs x and that of one eraser be Rs y.
It is given that Rama purchased 2 pencils and 3 erasers for Rs 9.
2x + 3y = 9
It is also given that Sonal purchased 4 pencils and 6 erasers for Rs 18.
4x + 6y = 18
Algebraic representation: The algebraic representation of the given situation is
2x + 3y = 9
4x + 6y = 18
Graphical representation: In order to obtain the graphical representation of the above pair of
linear equations, we find two points on the line representing each equation. That is, we find
two solutions of each equation. Let us find these solutions. We will try to find solutions
having integral values.
We have,
2x + 3y = 9
Putting x = 3, we get
6 + 3y = 9 3y = 15 y=5
Putting x = 0, we get
0 + 3y = 9 y=3
Thus, two solutions of 2x + 3y = 9 are:
x 3 0
y 5 3
We have,
4x + 6y = 18
Putting x = 3, we get
12 + 6y = 18 6y = 6 y=1
Putting x = 6, we get

Material downloaded from http://myCBSEguide.com and http://onlineteachers.co.in


Portal for CBSE Notes, Test Papers, Sample Papers, Tips and Tricks
24 + 6y = 18 6y = 42 y=7
Thus, two solutions of 4x + 6y = 18 are:
x 3 6
y 1 7
Now, we plot the points A(3, 5) and B(0, 3) and draw the line passing through these points
to obtain the graph of the line 2x + 3y = 9. Points P(3, 1) and Q(6, 7) are plotted on the graph
paper and we join them to obtain the graph of the line 4x + 6y = 18. We find that both the
lines AB and PQ coincide.

26. Solution:
Given: A ABC in which AD is a median.
2
1
To prove: AB2 + AC2 = 2AD2 + 2 BC or AB2 + AC2 = 2(AD2 + BD2)
2
Construction: Draw AE BC
Proof: Since AED = 90, therefore, in ADE, we have
ADE < 90 ADB > 90
Thus, ADB is an obtuse-angled triangle and ADC is an
acute-angled triangle.

ADB is an obtuse-angled triangle at D and AE BD


produced. Therefore, we have

Material downloaded from http://myCBSEguide.com and http://onlineteachers.co.in


Portal for CBSE Notes, Test Papers, Sample Papers, Tips and Tricks
AB2 = AD2 + BD2 + 2BD DE (i)
ACD is an acute-angled triangle at D and AE CD. Therefore, we have
AC2 = AD2 + DC2 2DC DE
AC2 = AD2 + BD2 2BD DE [ CD = BD] (ii)
Adding equations (i) and (ii), we get
AB2 + AC2 = 2(AD2 + BD2)
2 BC 2
AB2 + AC2 = 2 AD +
2
2
1
= 2AD + 2 BC
2

2
= 2AD2 + 2BD2
AB2 + AC2 = 2(AD2 + BD2)
27. Solution:
We have,
AB = a
AD + DB = a
AD + AD = a
a
2AD = a AD =
2
a
Thus, AD = DB =
2
By Pythagoras theorem, we have
AC2 = AB2 + BC2
b2 = a2 + BC2
BC2 = b2 a2

BC2 = b2 a2
Thus, in BCD, we have
a
Base = BC = b2 a2 and perpendicular = BD =
2
Applying Pythagoras theorem in BCD, we have
BC2 + BD2 = CD2

Material downloaded from http://myCBSEguide.com and http://onlineteachers.co.in


Portal for CBSE Notes, Test Papers, Sample Papers, Tips and Tricks
( )
2
2
a
b a
2 2
+ = CD2
2
a2
CD2 = b2 a2 +
4
4b2 4a2 + a2
CD2 =
4
4b2 3a2
CD2 =
4

4b2 3a2
CD =
2
a
BD 2 a
Now, sin = = =
CD 4b2 3a2 4b2 3a2
2

BC b2 a 2 2 b2 a 2
And, cos = = =
CD 4b2 3a2 4b2 3a2
2
2
2 b2 a2
2
a
Thus, sin2 + cos2 = +
4b 3a 4b 3a
2 2 2 2

= 2
a2
+
4 b2 a2 ( )
4b 3a2 4b2 3a2
a2 + 4b2 4a2
=
4b2 3a2
4b2 3a2
= =1
4b2 3a2
28. Solution:
We have,
cos A sin A
LHS = +
1 tan A 1 cot A
cos A sin A
= +
sin A cos A
1 1
cos A sin A

Material downloaded from http://myCBSEguide.com and http://onlineteachers.co.in


Portal for CBSE Notes, Test Papers, Sample Papers, Tips and Tricks
cos A sin A
= +
cos A sin A sin A cos A
cos A sin A
cos2 A sin2 A
= +
cos A sin A sin A cos A
cos2 A sin2 A
=
cos A sin A cos A sin A
cos2 A sin2 A
=
cos A sin A

=
( cos A sin A )( cos A+sin A )
cos A sin A
= cosA + sinA
= RHS
29. Solution:
Here, we are given the mid-values. So, should first find the upper and lower limits of the
various classes. The difference between two consecutive values is h = 125 115 = 10.
h
Lower limit of a class = Mid-value
2
h
Upper limit of a class = Mid-value +
2
Calculation of Median
Mid-value Class groups Frequency Cumulative frequency
115 110-120 6 6
125 120-130 25 31
135 130-140 48 79
145 140-150 72 151
155 150-160 116 267
165 160-170 60 327
175 170-180 38 365
185 180-190 22 387
195 190-200 3 390
N = fi = 390
We have,

Material downloaded from http://myCBSEguide.com and http://onlineteachers.co.in


Portal for CBSE Notes, Test Papers, Sample Papers, Tips and Tricks
N 390
N = 390 = = 195
2 2
N
The cumulative frequency just greater than , i.e., 195 is 267 and the corresponding class is
2
150-160. So, 150-160 is the median class.
Now,
N
F
Median = l + 2 h
f
195 151
= 150 + 10 = 153.80
116
30. Solution:
We have,
p( x ) = 3x 3 5x 2 11x 3

p(3) = 3 33 5 32 11 3 3
= 81 45 33 3
=0

p( 1) = 3 ( 1) 5 ( 1) 11 ( 1 ) 3
3 2

= 3 5 + 11 3
=0
3 2
1 1 1 1
p = 3 5 11 3
3 3 3 3
1 5 11
= + 3
9 9 3
=0
1
So, 3, 1 and are zeros of polynomial p(x).
3
1
Let = 3, = 1 and = . Then,
3
1 5 5 Coefficient of x 2
+ + = 3 1 = = =
3 3 3 Coefficient of x 3

1 1
+ + = 3 ( 1 ) + ( 1) + 3
3 3

Material downloaded from http://myCBSEguide.com and http://onlineteachers.co.in


Portal for CBSE Notes, Test Papers, Sample Papers, Tips and Tricks
1 11 Coefficient of x
= 3 + 1 = =
3 3 Coefficient of x 3

1 3 Constant term
= 3 ( 1) = 1 = =
3 3 Coefficient of x 3
31. Solution:
a. Price of petrol in December 2010 = x
Price of petrol in December 2014 = y
Price of petrol increased in 1 year = 4 2 = Rs 8
Price of petrol increased in 4 years (December 2010- December 2014) = 8 4 = Rs 32
Equation representing the price of petrol in December 2014 = y = x + 32
b. Price of CNG in December 2010 = a
Price of CNG in December 2014 =b
Price of CNG increased in 1 year = Rs 3
Price of CNG increased in 4 years (December 2010- December 2014) = 3 4 = Rs 12
Equation representing the price of CNG in December 2014 = b = a + 12
c. The value depicted by using CNG over petrol is environmental protection.

Material downloaded from http://myCBSEguide.com and http://onlineteachers.co.in


Portal for CBSE Notes, Test Papers, Sample Papers, Tips and Tricks
CBSE Sample Paper -05 (solved)
SUMMATIVE ASSESSMENT I
Class X Mathematics

Time allowed: 3 hours Maximum Marks: 90

General Instructions:

a) All questions are compulsory.


b) The question paper comprises of 31 questions divided into four sections A, B, C and D. You
are to attempt all the four sections.
c) Questions 1 to 4 in section A are one mark questions. These are MCQs. Choose the correct
option.
d) Questions 5 to 10 in section B are two marks questions.
e) Questions 11 to 20 in section C are three marks questions.
f) Questions 21 to 31 in section D are four marks questions.
g) There is no overall choice in the question paper. Use of calculators is not permitted.

SECTION A
1. Explain why 7 11 13 + 13 is a composite number.
2. Find whether the given system of equations has a unique solution, no solution or infinitely
many solutions:
x + y = 3, 2x + 5y = 12

sin18
3. Evaluate .
cos72
4. Find the mode of the following data:
120, 110, 130, 110, 120, 140, 130, 120, 140, 120
5. The perimeters of two similar triangles are 30 cm and 20 cm. If one side of the first triangle is
12 cm, determine the corresponding side of the second triangle.

SECTION B
6. Prove that the polynomial x 2 + 2x + 5 has no zero.
7. The areas of two similar triangles ABC and PQR are 64 cm2 and 121 cm2 respectively. If QR =
15.4 cm, find BC.
8. For any positive real number x, prove that there exists an irrational number y such that
0 < y < x.
9. Given that sin(A + B) = sinAcosB + cosAsinB, find the value of sin75.
10. Find the values of and for which the following system of linear equations has infinite
number of solutions.
2x + 3y = 7, 2x + ( + )y = 28

Material downloaded from http://myCBSEguide.com and http://onlineteachers.co.in


Portal for CBSE Notes, Test Papers, Sample Papers, Tips and Tricks
SECTION C
11. Find the largest positive integer that will divide 398, 436 and 542 leaving remainders 7, 11
and 15 respectively.
12. Find the condition that the zeros of the polynomial f ( x ) = x 3 px 2 + qx r may be in
arithmetic progression.
13. ABC is a right-angled triangle right angled at A. A circle is inscribed in it the lengths of two
sides containing the right angle are 6 cm and 8 cm. Find the radius of the circle.

14. Find the four angles of a cyclic quadrilateral ABCD in which A = (2x 5), B = (y + 5), C
= (2y + 15) and D = (4x 7).
15. A student noted the number of cars passing through a spot on a road for 100 periods each of
3 minutes and summarised it in the table given below. Find the mode of the data.
Number 0-10 10-20 20-30 30-40 40-50 50-60 60-70 70-80
of cars
Frequency 7 14 13 12 20 11 15 8

16. In a ABC, right angled at B, if AB = 4 and BC = 3, find all the six trigonometric ratios of A.
17. ABC is an isosceles triangle right-angled at B. Similar triangles ACD and ABE are constructed
on sides AC and AB. Find the ratio between the areas of ABE and ACD.

Material downloaded from http://myCBSEguide.com and http://onlineteachers.co.in


Portal for CBSE Notes, Test Papers, Sample Papers, Tips and Tricks
18. I am 3 times as old as my son. 5 years later, I shall be two and a half times as old as my son.
How old am I and how old is my son?

1 sin
19. Prove = sec tan .
1 + sin
20. Let ABC be a triangle and D and E be two points on side AB such that AD = BE. If DP || BC and
EQ || AC, then prove that PQ || AB.

SECTION D
21. The denominator of a fraction is 4 more than twice the numerator. When both the numerator
and denominator are decreased by 6, then the denominator becomes 12 times the
numerator. Determine the fraction.
1 sin A
22. If cosecA = 2, find the value of + .
tan A 1 + cos A
23. If x sin3 + y cos3 = sincos and x sin = y cos , prove that x 2 + y 2 = 1 .
24. A frequency distribution of the life times of 400 T.V. picture tubes tested in a company is
given below. Find the average life of a tube.
Life time (in hours) Frequency Life time (in hours) Frequency
300-399 14 800-899 62
400-499 46 900-999 48
500-599 58 1000-1099 22
600-699 76 1100-1199 6
700-799 68

25. What must be added to f ( x ) = 4 x 4 + 2x 3 2x 2 + x 1 so that the resulting polynomial is

divisible by g ( x ) = x 2 + 2x 3 ?

Material downloaded from http://myCBSEguide.com and http://onlineteachers.co.in


Portal for CBSE Notes, Test Papers, Sample Papers, Tips and Tricks
26. In trapezium ABCD, AB || DC and DC = 2AB. A line EF drawn parallel to AB cuts AD in F and
BE 3
BC in E such that = . Diagonal DB intersects EF at G. Prove that 7FE = 10AB.
EC 4

27. Solve the following system of linear equations graphically.


xy=1
2x + y = 8
Shade the area bounded by these two lines and y-axis. Also, determine this area.
28. Prove that the internal bisector of an angle of a triangle divides the opposite side internally in
the ratio of the sides containing the angle.
29. Following is the age distribution of a group of students. Draw the cumulative frequency
polygon, cumulative frequency curve (less than type) and hence obtain the median value.
Age Frequency Age Frequency
5-6 40 11-12 92
6-7 56 12-13 80
7-8 60 13-14 64
8-9 66 14-15 44
9-10 84 15-16 20
10-11 96 16-17 8

30. Prove
(1 + cot A + tan A )( sin A cos A ) = sin2 A cos2 A
sec3 A cosec3 A
31. In a housing society, people decided to do rainwater harvesting. Rainwater is collected in the
underground tank at the rate of 30 cm3/sec. Taking volume of water collected in x seconds as
y cm3.
a. Form a linear equation.
b. Write it in standard form as ax + by + c = 0.
c. Which values are promoted by the members of this society?

Material downloaded from http://myCBSEguide.com and http://onlineteachers.co.in


Portal for CBSE Notes, Test Papers, Sample Papers, Tips and Tricks
CBSE Sample Paper -05 (solved)
SUMMATIVE ASSESSMENT I
Class X Mathematics

Time allowed: 3 hours ANSWERS Maximum Marks: 90

SECTION A
1. Solution:
7 11 13 + 13 = 1001 + 13 = 1014
1014 = 2 3 13 13
i.e., it is the product of prime factors.
7 11 13 + 13 is a composite number.
2. Solution:
Here, a1 = 1, a2 = 2, b1 = 1, b2 = 5, c1 = 3 and c2 = 12
We have,
a1 1 b1 1
= and =
a2 2 b2 5
a1 b1
Since
a2 b2
The system has a unique solution.

3. Solution:
sin18 sin18
= [Using cos = sin(90 )]
cos72 cos ( 90 72 )
sin18
=
cos18
=1
4. Solution:
Let us first form the frequency table for the given data as given below:

Value (xi) 110 120 130 140


Frequency (fi) 2 4 2 2
We observe that the value 120 has the maximum frequency.
Thus, the mode is 120.

5. Solution:
Let ABC and DEF be two similar triangles of perimeters P1 and P2 respectively. Also, let AB
= 12 cm, P1 = 30 cm and P2 = 20 cm. Then,
AB BC AC P1
= = =
DE EF DF P2
AB P1
=
DE P2
12 30 12 20
= DE = = 8 cm
DE 20 30

Material downloaded from http://myCBSEguide.com and http://onlineteachers.co.in


Portal for CBSE Notes, Test Papers, Sample Papers, Tips and Tricks
Thus, the corresponding side of the second triangle is 8 cm.

SECTION B
6. Solution:
Let f(x) = x 2 + 2x + 5
= x 2 + 2x + 1 + 4

= ( x + 1) + 4
2

Now, for every real value of x, ( x + 1 ) 0


2

For every real value of x, ( x + 1 ) + 4 4


2

For every real value of x, f ( x ) 4 and hence it has no zero.

7. Solution:
Given that ABC PQR
ar ( ABC ) BC2
=
ar ( PQR ) QR 2

64 BC2
=
121 (15.4 )2

8 15.4
= 11.2 cm
11
8 BC
BC = =
11 15.4
8. Solution:
x
If x is irrational, then y = is also an irrational number such that 0 < y < x.
2
x x
If x is rational, then is an irrational number such that < x as 2 > 1.
2 2
x
y= is an irrational number such that 0 < y < x.
2
9. Solution:
Putting A = 45 and B = 30 in sin(A + B) = sinAcosB + cosAsinB, we get
sin(45 + 30) = sin45cos30 + cos45sin30

Material downloaded from http://myCBSEguide.com and http://onlineteachers.co.in


Portal for CBSE Notes, Test Papers, Sample Papers, Tips and Tricks
1 3 1 1
sin75 = +
2 2 2 2

3 1 3 +1
= + =
2 2 2 2 2 2
10. Solution:
The given system of equations will have infinite number of solutions, if
2 3 7
= =
2 + 28
1 3 1
= =
+ 4
1 1 3 1
= and =
4 + 4

= 4 and + = 12
= 4 and = 12 4 = 8
SECTION C
11. Solution:
It is given that on dividing 398 by the required number, there is a remainder of 7. This means
that 398 7 = 391 is exactly divisible by the required number. In other words, required
number is a factor of 391.
Similarly, required positive integer is a factor of 436 11 = 425 and 542 15 = 527.
Clearly, required number is the HCF of 391, 425 and 527.
Using the factor tree, the prime factorisations of 391, 425 and 527 are as follows:
391 = 17 23
425 = 52 17
527 = 17 31
HCF of 391, 425 and 527 is 17.
Thus, the required number is 17.
12. Solution:
Let a d, a and a + d be the zeros of the polynomial f(x). Then,
Coefficient of x 2
Sum of the zeros =
Coefficient of x 3

Material downloaded from http://myCBSEguide.com and http://onlineteachers.co.in


Portal for CBSE Notes, Test Papers, Sample Papers, Tips and Tricks
( p)
(a d ) + a + (a + d ) =
1
p
3a = p a=
3
Since a is a zero of the polynomial f(x). Therefore,
f(a) = 0
a3 pa2 + qa r = 0
3 2
p p p p
p + q r = 0 a = 3
3 3 3
p3 3p3 + 9pq 27r = 0

2p3 9pq + 27r = 0


13. Solution
Using Pythagoras theorem in BAC, we have
BC2 = AB2 + AC2
BC2 = 62 + 82 = 100
BC = 10 cm
Now,

Area of ABC = Area of OAB + Area of OBC + Area of OCA


1 1 1 1
AB AC = AB r + BC r + CA r
2 2 2 2
1 1 1 1
6 8 = 6 r + 10 r + 8 r
2 2 2 2
48 = 24r
r = 2 cm
14. Solution:
We know that the sum of the opposite angles of a cyclic quadrilateral is 180. In the cyclic
quadrilateral ABCD, angles A and C and angles B and D form pairs of opposite angles.
A + C = 180 and B + D = 180
2x 1 + 2y + 15 = 180 and y + 5 + 4x 7 = 180
2x + 2y = 166 and 4x + y = 182

Material downloaded from http://myCBSEguide.com and http://onlineteachers.co.in


Portal for CBSE Notes, Test Papers, Sample Papers, Tips and Tricks
x + y = 83 (i)
And, 4x + y = 182 (ii)
Subtracting equation (i) from equation (ii), we get
3x = 99 x = 33
Substituting x = 33 in equation (i), we get y = 50
Hence, A = ( 2x 1) = ( 2 33 1) = 65 , B = ( y + 5) = (50 + 5) = 55

C = ( 2 y + 15) = ( 2 50 + 15) = 115 and D = ( 4 x 7 ) = ( 4 33 7 ) = 125

15. Solution:
The modal class is 40-50, since it has the maximum frequency.
l = 40, f1 = 20, f0 = 12, f2 = 11, h = 10
f1 f0
Mode = l + h
2 f1 f 0 f2
20 12
= 40 + 10
2 20 12 11
8
= 40 + 10
17
= 40 + 4.71
= 44.71 cars
16. Solution :
We have AB = 4 and BC = 3
By Pythagoras theorem, we have
AC2 = AB2 + BC2

AC = AB2 + BC2

AC = 42 + 32

AC = 25 = 5
When we consider the t-ratios of A, we have
Base = AB = 4, Perpendicular = BC = 3 and Hypotenuse = AC = 5
BC 3 AB 4 BC 3
sinA = = , cosA = = , tanA = =
AC 5 AC 5 AB 4
AC 5 AC 5 AB 4
cosecA = = , secA = = and cotA = =
BC 3 AB 4 BC 3

Material downloaded from http://myCBSEguide.com and http://onlineteachers.co.in


Portal for CBSE Notes, Test Papers, Sample Papers, Tips and Tricks
17. Solution:
Let AB = BC = x.
It is given that ABC is a right-angled at B.
AC2 = AB2 + BC2
AC2 = x2 + x2
AC = 2x
It is given that
ABE ACD
Area ( ABE ) AB2
=
Area ( ACD ) AC2

x2
=
( )
2
2x

1
=
2
18. Solution
Suppose my age is x years and my sons age is y years. Then,
x = 3y (i)
5 years later, my age will be (x + 5) years and my sons age will be (y + 5) years.
5
x+5= ( y + 5)
2
2x 5y 15 = 0 (ii)
Putting x = 3y in equation (ii), we get
6y 5y 15 = 0 y = 15
Putting y = 15 in equation (i), we get
x = 45
19. Solution
We have,

1 sin
LHS =
1 + sin

1 sin 1 sin
=
1 + sin 1 sin

Material downloaded from http://myCBSEguide.com and http://onlineteachers.co.in


Portal for CBSE Notes, Test Papers, Sample Papers, Tips and Tricks
(1 sin)
2

=
1 sin2

(1 sin)
2

=
cos2
2
1 sin
= cos

1 sin
=
cos
1 sin
= = sec tan = RHS
cos cos
20. Solution
In ABC, we have
DP || BC and EQ || AC
AD AP BE BQ
= and =
DB PC EA QC
AD AP AD BQ
= and = [ EA = ED + DA = ED + BE = BD, AD = BE]
DB PC DB QC
AP BQ
=
PC QC

In a ABC, P and Q divide sides CA and CB respectively in the same ratio.


PQ || AB.
SECTION D
21. Solution
Let the numerator and denominator of the fraction be x and y respectively.
Then,
x
Fraction =
y
It is given that
Denominator = 2(Numerator) + 4
y = 2x + 4
2x y + 4 = 0
According to the given condition, we have

Material downloaded from http://myCBSEguide.com and http://onlineteachers.co.in


Portal for CBSE Notes, Test Papers, Sample Papers, Tips and Tricks
y 6 = 12(x 6)
y 6 = 12x 72
12x y 66 = 0
Thus, we have the following system of equations
2x y + 4 = 0 (i)
12x y 66 = 0 (ii)
Subtracting equation (i) from equation (ii), we get
10x 70 = 0
x=7
Putting x = 7 in equation (i), we get
14 y + 4 = 0
y = 18
7
Hence, required fraction =
18
22. Solution:
We have,
Hypotenuse 2
cosecA = =
Perpendicular 1
So, we draw a right triangle, right angled at B such that
Perpendicular = BC = 1, Hypotenuse = AC = 2
By Pythagoras theorem, we have
AC2 = AB2 + BC2
22 = AB2 + 12
AB2 = 4 1
AB2 = 3

AB = 3
Now,
BC 1 BC 1 AB 3
tanA = = , sinA = = and cosA = =
AB 3 AC 2 AC 2

Material downloaded from http://myCBSEguide.com and http://onlineteachers.co.in


Portal for CBSE Notes, Test Papers, Sample Papers, Tips and Tricks
1
1 sin A 1
+ = + 2
tan A 1 + cos A 1 3
1+
3 2
1
3
= + 2
1 2+ 3
2

3 1
= +
1 2+ 3

1 2 3
= 3+
2+ 3 2 3

2 3
= 3+
( 3)
2
22

2 3
= 3+
4 3

(
= 3 + 2 3 = 2 )
23. Solution:
We have,
x sin3 + y cos3 = sincos

( ) ( )
x sin sin2 + y cos cos2 = sincos

( ) ( )
x sin sin2 + x sin cos2 = sincos [ x sin = y cos]
( )
x sin sin2 +cos2 = sincos

x sin = sincos
sincos
x= = cos
sin
Now,
x sin = y cos

cossin = y cos [ x = cos]


cossin
y= = sin
cos

Material downloaded from http://myCBSEguide.com and http://onlineteachers.co.in


Portal for CBSE Notes, Test Papers, Sample Papers, Tips and Tricks
x 2 + y 2 = sin2 + cos2 = 1
24. Solution:
Here, the class intervals are formed by exclusive method. If we make the series an inclusive,
one of the mid-values remain same. So, there is no need to convert the series into an
inclusive form.
Let the assumed mean be A = 749.5 and h = 100.
Calculation of mean
Life time Frequency Mid-values di = xi A xi A fiui
ui =
(in hours) fi xi = xi 749.5 h
xi 749.5
ui =
100
300-399 14 349.5 400 4 56
400-499 46 449.5 300 3 138
500-599 58 549.5 200 2 116
600-699 76 649.5 100 1 76
700-799 68 749.5 0 0 0
800-899 62 849.5 100 1 62
900-999 48 949.5 200 2 96
1000-1099 22 1049.5 300 3 66
1100-1199 6 1149.5 400 4 24
N = fi = 400 f u = 138
i i

We have N = 400, A = 749.5, h = 100 and f u = 138


i i

1
X = A + h fi ui
N
138
= 749.5 + 100
400
138
= 749.5
4
= 749.5 34.5 = 715
Thus, the average life time of a tube is 715 hours.

Material downloaded from http://myCBSEguide.com and http://onlineteachers.co.in


Portal for CBSE Notes, Test Papers, Sample Papers, Tips and Tricks
25. Solution:
By division algorithm, we have
f ( x ) = g( x ) q( x ) + r ( x )

f ( x ) r ( x ) = g( x ) q( x )

f ( x ) + {r ( x )} = g ( x ) q ( x )

Clearly, RHS is divisible by g(x). Therefore, LHS is also divisible by g(x). Thus, if we add r(x)
to f(x), then the resulting polynomial is divisible by g(x). Let us now find the remainder when
f(x) is divided by g(x).
4 x 2 6 x + 22
x 2 + 2 x 3 4 x 4 + 2 x 3 2x 2 + x 1
4 x 4 + 8 x 3 12x 2
+
6 x 3 + 10x 2 + x 1
6 x 3 12x 2 + 18 x
+ +
22x 2 17 x 2
22x 2 + 44 x 66
+
61x + 65

r(x) = 61x + 65
Thus, we should add r(x) = 61x 65 to f(x) so that the resulting polynomial is divisible by
g(x).

26. Solution:
In DFG and DAB, we have
1 = 2 [ AB || DC || EF, 1 and 2 are corresponding angles]
FDG = ADB [Common]
So, by AA-criterion of similarity, we have
DF FG
DFG DAB = (i)
DA AB
In trapezium ABCD, we have
EF || AB || DC

Material downloaded from http://myCBSEguide.com and http://onlineteachers.co.in


Portal for CBSE Notes, Test Papers, Sample Papers, Tips and Tricks
AF BE
=
DF EC
AF 3 BE 3
= EC = 4 ( Given )
DF 4
AF 3
+1 = +1 [Adding 1 on both sides]
DF 4
AF + DF 7
=
DF 4
AD 7 DF 4
= = (ii)
DF 4 AD 7
From (i) and (ii), we get
FG 4 4
= FG = AB (iii)
AB 7 7
In BEG and BCD, we have
BEG = BCD [Corresponding angles]
B = B [Common]
BEG BCD [By AA-criterion of similarity]
BE EG
=
EC CD
3 EG BE 3 EC 4 EC 4 BC 7
= = =
EC 4 BE 3 BE + 1 = + 1 =
BE 3

7 CD 3
3 3 6
EG = CD = 2AB = AB (iv)
7 7 7
Adding (iii) and (iv), we get
4 6
FG + EG = AB + AB
7 7
10
EF = AB
7
7EF = 10AB
27. Solution:
We have,
xy=1
2x + y = 8

Material downloaded from http://myCBSEguide.com and http://onlineteachers.co.in


Portal for CBSE Notes, Test Papers, Sample Papers, Tips and Tricks
Graph of the equation x y = 1:
We have,
xy=1 y = x 1 and x = y + 1
Putting x = 0, we get y = 1
Putting y = 0, we get x = 1
Thus, we have the following table for the points on the line x y = 1:

x 0 1
y 1 0

Graph of the equation 2x + y = 8:


We have,
8 y
2x + y = 8 y = 8 2x and x =
2
Putting x = 0, we get y = 8
Putting y = 0, we get x = 4
Thus, we have the following table for the points on the line 2x + y = 8:

x 0 8
y 8 0

Plotting points A(0, 1), B(1, 0) on the graph paper and drawing a line passing through them,
we obtain the graph of the line represented by the equation x y = 1.

Material downloaded from http://myCBSEguide.com and http://onlineteachers.co.in


Portal for CBSE Notes, Test Papers, Sample Papers, Tips and Tricks
Plotting points C(0, 8), D(4, 0) on the same graph paper and drawing a line passing through
them, we obtain the graph of the line represented by the equation 2x + y = 8.
Clearly, the two lines intersect at P(3, 2). The area enclosed by the lines represented by the
given equations and the y-axis is shaded.
Now, required area = Area of the shaded region
= Area of PAC
1
= ( Base Height )
2
1
= ( AC PM ) [ PM = x-coordinate of P = 3]
2
1
= ( 9 3) = 13.5 sq. units
2
28. Solution:
Given: A ABC in which AD is the internal bisector of A and meets BC in D.
BD AB
To prove: =
DC AC

Construction: Draw CE || DA to meet BA produced in E.


Proof: Since CE || DA and AC cuts them,
2 = 3 [Alternate angles] (i)
And, 1 = 4 [Corresponding angles] (ii)
But, 1 = 2 [ AD is the bisector of A]
From (i) and (ii), we get
3 = 4

Material downloaded from http://myCBSEguide.com and http://onlineteachers.co.in


Portal for CBSE Notes, Test Papers, Sample Papers, Tips and Tricks
Thus, in ACE, we have
3 = 4
AE = AC [Sides opposite to equal angles are equal] (iii)
Now, in BCE, we have
DA || CE
BD BA
= [Using Basic Proportionality Theorem]
DC AE
BD AB
= [ BA = AB and AE = AC (From (iii)]
DC AC
BD AB
Thus, =
DC AC
29. Solution:
We first prepare the cumulative frequency table by less than method as given below:
Age Frequency Age less than Cumulative frequency
5-6 10 6 40
6-7 56 7 96
7-8 60 8 156
8-9 66 9 222
9-10 84 10 306
10-11 96 11 402
11-12 92 12 494
12-13 80 13 574
13-14 64 14 638
14-15 44 15 682
15-16 20 16 702
16-17 8 17 710

Other than the given class intervals, we assume a class 4-5 before the first class-interval 5-6
with zero frequency.
Now, we mark the upper class limits (including the imagined class) along x-axis on a suitable
scale and the cumulative frequencies along y-axis on a suitable scale.

Material downloaded from http://myCBSEguide.com and http://onlineteachers.co.in


Portal for CBSE Notes, Test Papers, Sample Papers, Tips and Tricks
Thus, we plot the points (5, 0), (6, 40), (7, 96), (8, 156), (9, 222), (10, 306), (11, 402), (12,
494), (13, 574), (14, 638), (15, 682), (16, 702) and (17, 710). These points are marked and
joined by line segments to obtain the cumulative frequency polygon as shown in the figure.

In order to obtain the cumulative frequency curve, we draw a smooth curve passing through
the points discussed above.
The graph given below shows the total number of students as 710. The median is the age
corresponding to
N 710
= = 355 students.
2 2
In order to find the
median, we first locate
the point corresponding
to 355th student on y-
axis. Let the point be P.
From this point, draw a
line parallel to the x-axis
cutting the curve at Q.
From this point Q, draw a line parallel to y-axis and meeting x-axis at the point M. The x-
coordinate of M is 10.5.
Hence, median is 10.5.

Material downloaded from http://myCBSEguide.com and http://onlineteachers.co.in


Portal for CBSE Notes, Test Papers, Sample Papers, Tips and Tricks
30. Solution:
We have,

LHS =
(1 + cot A + tan A )( sin A cos A )
sec3 A cosec3 A
cos A sin A
1 + + ( sin A cos A )
= sin A cos A
1 1
3
3
cos A sin A
cos2 A + sin2 A
1 + ( sin A cos A )
sin A cos A
=
sin3 A cos3 A
3 3
sin A cos A
sin A cos A + cos2 A + sin2 A sin3 A cos3 A
= 3 ( sin A cos A)
sin A cos A sin A cos3 A
sin A cos A + 1
=
sin A cos A
3 3 ( )
sin2 A cos2 A ( sin A cos A )

( sin A cos A + 1) ( sin2 A cos2 A ) ( sin A cos A )


=
( sin A cos A ) ( sin2 A + cos2 A + sin A cos A ) ( )
a3 b3 = ( a b ) a2 + b2 + ab

=
( sin A cos A + 1) sin2 A cos2 A
1 + sin A cos A
= sin2 A cos2 A = RHS
31. Solution:
Rate at which rainwater is collected in the tank = 30 cm3/sec
Time for which water is collected = x seconds
Total amount of water collected = y cm3
a. According to the given condition, linear equation formed is y = 30x
b. The equation in standard form is 30x y + 0 = 0
c. Values promoted by the members of the society are environmental protection and co-
operation.

Material downloaded from http://myCBSEguide.com and http://onlineteachers.co.in


Portal for CBSE Notes, Test Papers, Sample Papers, Tips and Tricks
CBSE Sample Paper -01 (unsolved)
SUMMATIVE ASSESSMENT I
Class X Mathematics

Time allowed: 3 hours Maximum Marks: 90

General Instructions:

a) All questions are compulsory.


b) The question paper comprises of 31 questions divided into four sections A, B, C and D. You
are to attempt all the four sections.
c) Questions 1 to 4 in section A are one mark questions. These are MCQs. Choose the correct
option.
d) Questions 5 to 10 in section B are two marks questions.
e) Questions 11 to 20 in section C are three marks questions.
f) Questions 21 to 31 in section D are four marks questions.
g) There is no overall choice in the question paper. Use of calculators is not permitted.

SECTION A

1. If the graph of quadratic polynomial ax2 + bx + c cuts negative direction of y-axis, then what is
the sign of c?
2. For what value of k, the following pair of linear equations has infinitely many solutions?
10x + 5y (k 5) = 0
20x + 10y k = 0
3. In the given figure, if AD is the bisector of A, what is AC?

4. If sin A + sin2 A = 1, then what will be the value of cos2 A + cos4 A?

Material downloaded from http://myCBSEguide.com and http://onlineteachers.co.in


Portal for CBSE Notes, Test Papers, Sample Papers, Tips and Tricks
SECTION B

5. Can (x 3) be the remainder on division of a polynomial p(x) by (2x + 3)? Justify your
answer.
6. Evaluate sin 39 cos 51.
7. Find the value of HCF LCM for the numbers 80 and 120.
8. Find the mode of the following distribution.
Class-interval: 010 1020 2030 3040 4050 5060 6070 70 80

Frequency: 5 8 7 12 28 20 10 10

9. If sec4A = cosec (A 20), where 4A is an acute angle, find the value of A.

10. In the given figure, PQ || CD and PR || CB. Prove that = .

SECTION C

11. Use Euclids division algorithm to find the HCF of 196 and 38220.
12. Prove that 5 + 3 is irrational.
13. If and are the zeroes of the polynomial x2 2x 15, then form a quadratic polynomial
whose zeroes are 2 and 2.
14. The diagonals of a trapezium ABCD with AB || DC intersect each other at point O. if AB = 2CD,
find the ratio of the areas of triangles AOB and COD.

Material downloaded from http://myCBSEguide.com and http://onlineteachers.co.in


Portal for CBSE Notes, Test Papers, Sample Papers, Tips and Tricks
15. The perimeters of two similar triangles are 25 cm and 15 cm, respectively. If one side of first
triangle is 9 cm, what is the corresponding side of the other triangle/

16. If tan = , find the value of .
1 + cos

17. The arithmetic mean of the following data is 14. Find the value of k.
x: 5 10 15 20 25
f: 7 k 8 4 5
18. There are two examination rooms A and B. If 10 candidates are sent from A to B, the number
of students in each room is same. If 20 candidates are sent from B to A, the number of
students in A is double the number of students in B. Find the number of students in each
room.
19. A vertical stick of length 6 m casts a shadow 4 m long on the ground and at the same time a
tower casts a shadow 28 m long. Find the height of the tower.
20. Find the mean of the following frequency distribution, using step-division method.
Class: 0 10 10 20 20 30 30 40 40 50

Frequency: 7 12 13 10 8

SECTION D

21. Solve the following system of equations:


12y + 30x = 5xy, 30y + 40x = 9xy (x 0, y 0)
22. Find the other zeroes of the polynomial p(x) = 2x4 + 7x3 19x2 14x + 30 if two of its zeroes
are 2 and 2.
23. Check graphically whether the pair of linear equation 4x y 8 = 0 and 2x 3y + 6 = 0 is
consistent. Also, find the vertices of the triangle formed by these lines with the x-axis.
24. In an isosceles triangle ABC with AB = AC and BD AC. Prove that BD2 CD2 = 2CD.AD.
25. Prove that in a triangle, if the square of one side is equal to the sum of the squares of the
other two sides, then the angles opposite to the first side is a right angle.
1 + sec sin
26. Prove = 1 - cos.
sec

27. If acos + bsin = m and asin - bcos = n, then prove that a2 + b2 = m2 + n2

28. Evaluate 4(sin430 + cos460 (sin260 cos245) + tan260.

Material downloaded from http://myCBSEguide.com and http://onlineteachers.co.in


Portal for CBSE Notes, Test Papers, Sample Papers, Tips and Tricks
29. If mean of the following distribution is 54, find the value of k.
Class: 0 20 20 40 40 60 60 80 80 100
Frequency: 7 k 10 9 13

30. Find the mean, median and mode of the following data:
Class: 0 50 50 100 100 150 150 200 200 250 250 300
Frequency: 2 3 5 6 5 3

31. Sonal has some pencils and pens which together are 70 in number. If she gives 5 pencils to
her younger sister and 5 pens to her younger brother, then the number of pencils would
become double of the number of pens. Find the original number of pens and pencils that
Sonal had. Also tell which value of Sonal is depicted in the question?

Material downloaded from http://myCBSEguide.com and http://onlineteachers.co.in


Portal for CBSE Notes, Test Papers, Sample Papers, Tips and Tricks
CBSE Sample Paper -02 (unsolved)
SUMMATIVE ASSESSMENT I
Class X Mathematics

Time allowed: 3 hours Maximum Marks: 90

General Instructions:

a) All questions are compulsory.


b) The question paper comprises of 31 questions divided into four sections A, B, C and D. You
are to attempt all the four sections.
c) Questions 1 to 4 in section A are one mark questions. These are MCQs. Choose the correct
option.
d) Questions 5 to 10 in section B are two marks questions.
e) Questions 11 to 20 in section C are three marks questions.
f) Questions 21 to 31 in section D are four marks questions.
g) There is no overall choice in the question paper. Use of calculators is not permitted.

SECTION A

1. If the product of two numbers is 1080 and their HCF is 30, find their LCM.
2. For what value of k, the following pair of linear equations has infinitely many solutions?
10x + 5y (k 5) = 0
20x + 10y k = 0
3. If tanA = and A + B = 90, then what is the value of cotB?

4. Find the class marks of the class 10 25.

SECTION B
3 sin tan 1
5. If is an acute angle such that cos = , then find .
4 2 tan 2
6. Prove that 3 2 is irrational.
7. If and are the zeroes of the polynomial 2x2 7x + 3, then find the value of 2 + 2.
8. Sum of two natural numbers is 21 and their difference is 11, find the numbers.
AD BE C
9. In the given figure, if = and CDE = CED, prove that ABC
DC EC D E
is isosceles.
A B
10. Solve 2cos = 1 for 0 < < 90.

Material downloaded from http://myCBSEguide.com and http://onlineteachers.co.in


Portal for CBSE Notes, Test Papers, Sample Papers, Tips and Tricks
SECTION C
11. Solve the following system of equations by using the method of cross multiplication:
2x + y 35 = 0
3x + 4y 65 = 0
12. Two containers contain 430 and 120 litres of milk, respectively. Find the maximum capacity
of a measuring vessel which can measure the milk contained in either of the containers in
exact number of times.
13. The numerator of a fraction is 4 less than its denominator. If the numerator is decreased by 1
then denominator is 8 times the numerator. Find the fraction.
1
14. X takes 3 hours more than Y to walk 30 km. But is X doubles his pace, he is ahead of Y by 1
2
hours. Find their speed of walking.
15. In ABC, right-angled at B. AB = 3 cm and AC = 6 cm. Find A and C.
16. Prove (sin8 cos8) = (sin2 cos2)(1 2sin2cos2)
1 2 1
17. Evaluate sin230 cos245 + 4tan230 + sin 90 - 2cos290 + .
2 24
18. If the mean of the following distribution is 6, find the value of p.
x: 2 4 6 10 p+5
f: 3 2 3 1 2
19. Prove that the line drawn from the mid-point of one side of a triangle parallel to another side
bisects the third side.
20. Determine whether the triangle having sides (a 1) cm, 2 a and (a + 1) cm is a right angled
triangle.

SECTION D
21. Show that any positive odd integer is of the form 6q + 1 or 6q + 3 or 6q + 5, where q is some
integer.
22. What must be subtracted from 8x4 + 14x3 2x2 + 7x 8 so that the resulting polynomial is
exactly divisible by 4x2 + 3x 2?
23. Determine graphically the vertices of a trapezium, the equations of whose sides are x = 0, y =
0, y = 4 and 2x + y = 6

Material downloaded from http://myCBSEguide.com and http://onlineteachers.co.in


Portal for CBSE Notes, Test Papers, Sample Papers, Tips and Tricks
24. If A be the area of a right triangle and b one of the sides containing the right angle, prove that
2 Ab
the length of the altitude on the hypotenuse is .
b + 4 A2
4

25. There is a staircase as shown in the given figure, connecting points A and B. Measurements of
steps are marked in the figure. Find the straight line distance between A and B.

26. If the median of the distribution given below is 28.5, find the value of x and y.
Class: 0 10 10 20 20 30 30 40 40 50 50 60
No. of 5 x 20 15 y 5
students:

tan A tan B + tan A cot B sin2 B


27. If A + B = 90, prove that = tanA.
sin A sec B cos2 A
2 2
28. If x = cot + tan and y = sec cos, prove that ( x 2 y ) 3 ( xy 2 ) 3
= 1.
29. Draw an ogive and the cumulative frequency polygon for the following frequency
distribution by less than method.
Marks: 0 10 10 20 20 30 30 40 40 50 50 60
Number of students 7 10 23 51 6 3

30. In an acute angled triangle ABC, if tan (A + B C) = 1 and sec (B + C A) = 2, find the value of
A, B and C.
31. Reena and Niti went to the market. Reena purchased 20 pencils and 5 erasers for Rs 55. Niti
also wanted to purchase some items but she did not have money. Reena gave some money to
Niti and Niti also purchased 10 pencils and 4 erasers of the same kind for Rs 32. Represent
the situation algebraically and graphically. Also tell which value of Reena is depicted in the
question?

Material downloaded from http://myCBSEguide.com and http://onlineteachers.co.in


Portal for CBSE Notes, Test Papers, Sample Papers, Tips and Tricks
CBSE Sample Paper -03 (unsolved)
SUMMATIVE ASSESSMENT I
Class X Mathematics

Time allowed: 3 hours Maximum Marks: 90

General Instructions:

a) All questions are compulsory.


b) The question paper comprises of 31 questions divided into four sections A, B, C and D. You
are to attempt all the four sections.
c) Questions 1 to 4 in section A are one mark questions. These are MCQs. Choose the correct
option.
d) Questions 5 to 10 in section B are two marks questions.
e) Questions 11 to 20 in section C are three marks questions.
f) Questions 21 to 31 in section D are four marks questions.
g) There is no overall choice in the question paper. Use of calculators is not permitted.

SECTION A
43
1. The decimal expansion of the rational number will terminate after how many places of
2 53
4

decimals?
2. The area of the triangle formed by the lines y = x, x = 6 and y = 0 is
a. 36 sq.units b. 18 sq.units c. 9 sq.units d. 72 sq.units
3. If am bl, then the system of equations
ax + by = c
lx + my = n
a. has a unique solution b. has no solution
c. has infinitely many solutions d. may or may not have a solution
4. If the arithmetic mean of x, x + 3, x + 6, x + 9 and x + 12 is 10, then x =
a. 1 b. 2 c. 6 d. 4

SECTION B
5. What can you say about the prime factorisation of the denominators of the rational number
27.142857 .
6. If (x + a) is a factor of 2x2 + 2ax + 5x + 10, find a.
7. If sec2 (1 + sin)(1 sin) = k, then find the value of k.

Material downloaded from http://myCBSEguide.com and http://onlineteachers.co.in


Portal for CBSE Notes, Test Papers, Sample Papers, Tips and Tricks
8. In a ABC, A = 90, AB = 5 cm and AC = 12 cm. If AD BC, then find AD.
EF EC
9. In the given figure, DE || AC and DF ||AE. Prove that = .
BF BE

10. Find the mode of the following frequency distribution:


Size of items: 0-4 4-8 8-12 12-16 16-20
Frequency: 5 7 9 17 12

SECTION C
r r sin s cos
11. If tan = , find the value of .
s r sin + s cos
12. In a triangle PQR, right angled at Q, PR + QR = 25 cm and PQ = 5 cm. Find the values of sinP,
cosP and tanP.
13. If the mean of the following distribution is 54, find the value of p.
Class 0-20 20-40 40-60 60-80 80-100
Frequency 7 p 10 9 13
14. Prove that the roots of the equation (a b + c)x2 + 2(a b)x + (a b c) = 0 are real.
2 4
15. Find the fraction which becomes when the numerator is increased by 2 and equal to
3 7
when the denominator is increased by 4.
16. In the following figure, find whether the triangles are similar. Give reasons in support of your
answer.

Material downloaded from http://myCBSEguide.com and http://onlineteachers.co.in


Portal for CBSE Notes, Test Papers, Sample Papers, Tips and Tricks
a 3 3 2
17. In an equilateral triangle with side a, prove that altitude is and area is a .
2 4
18. Check whether the first polynomial is a factor of the second polynomial by applying the
division algorithm.
t2 3; 2t4 + 3t3 2t2 9t - 12
19. If sin + cos = 2 sin(90 ), determine cot.
20. Find the mean marks of the students from the following cumulative frequency distribution:
Marks Below Below Below Below Below Below Below Below Below Below
10 20 30 40 50 60 70 80 90 100
No. of 5 9 17 29 45 60 70 78 83 85
students

SECTION D
21. Form the pair of linear equations in the following problem and find its solution graphically.
20 students of a class took part in Science quiz. If the number of girls is 6 more than boys,
find the number of boys and girls who took part in the quiz.
22. The median of the following data is 525. Find the values of x and y, if the total frequency is
100.
Class 0-100 100- 200- 300- 400- 500- 600- 700- 800- 900-
interval 200 300 400 500 600 700 800 900 1000
Frequency 2 5 x 12 17 20 y 9 7 4

1 1
23. If sec = x + , prove that sec + tan = 2x or .
4x 2x
24. The following table shows the ages of the patients admitted in a hospital during a year:
Age (in years) 5-15 15-25 25-35 35-45 45-55 55-65
No. of students 6 11 21 23 14 5
Find the mode and the mean of the data given above. Compare and interpret the two
measures of central tendency.
25. If the polynomial f(x) = x4 6x3 + 16x2 25x + 10 is divided by another polynomial x2 2x + k,
the remainder comes out be to be x + a. Find the value of k and a.

Material downloaded from http://myCBSEguide.com and http://onlineteachers.co.in


Portal for CBSE Notes, Test Papers, Sample Papers, Tips and Tricks
26. If cos + cos2 = 1, prove that
sin12 + 3sin10 + 3sin8 + sin6 + 2sin4 + 2sin2 2 = 1
27. The perpendicular AD on the base BC of a ABC intersects BC at D so that DB = 3CD. Prove
that 2AB2 = 2AC2 + BC2.
28. Prove that in an equilateral triangle, three times the square of a side is equal to four times the
square of its altitudes.
29. If 2 + 45 and 30 are acute angles, find the degree measure of satisfying
Sin(2 + 45) = cos(30 ).
30. Solve the following system of equations graphically
x + 3y = 6
2x 3y = 12
and hence find the value of a, if 4x + 3y = a.
31. Raj, on his birthday, distributed chocolates in an orphanage. He gave 2 chocolates to each
child and 20 chocolates to adults. Considering number of children as x and total number of
chocolates distributed is y, form the linear equation. If the total number of chocolates
distributed is 150, how many children are there in the orphanage. Explain the value depicted
by Raj in the question.

Material downloaded from http://myCBSEguide.com and http://onlineteachers.co.in


Portal for CBSE Notes, Test Papers, Sample Papers, Tips and Tricks
CBSE Sample Paper -04 (unsolved)
SUMMATIVE ASSESSMENT I
Class X Mathematics

Time allowed: 3 hours Maximum Marks: 90

General Instructions:

a) All questions are compulsory.


b) The question paper comprises of 31 questions divided into four sections A, B, C and D. You
are to attempt all the four sections.
c) Questions 1 to 4 in section A are one mark questions. These are MCQs. Choose the correct
option.
d) Questions 5 to 10 in section B are two marks questions.
e) Questions 11 to 20 in section C are three marks questions.
f) Questions 21 to 31 in section D are four marks questions.
g) There is no overall choice in the question paper. Use of calculators is not permitted.

SECTION A
1. If two positive integers a and b are expressible in the form a = pq3 and b = p2q; p and q being
prime numbers, then LCM (a, b) is
a. pq b. p2q3 c. p3q3 d. p3q2
2. If the sum of the zeros of the quadratic polynomial f(x) = x2 4x + k is 3, find the value of k.

A
5 cm 4 cm
3. In the adjoining figure, find AC. D E
10 cm
B C

4. Sin2A = 2sinA is true when A =


a. 0 b. 30 c. 45 d. 60

SECTION B
1
5. Find a quadratic polynomial with 2 and as the sum and product of its zeros,
3
respectively.
6. Solve the following system of equations by using the method of substitution.
3x 5y = 1; x y = 1

Material downloaded from http://myCBSEguide.com and http://onlineteachers.co.in


Portal for CBSE Notes, Test Papers, Sample Papers, Tips and Tricks
7. The perimeters of two similar triangles ABC and PQR are respectively 24 cm and 12 cm. If PQ
= 12 cm, find AB.
8. In a triangle ABC, right angled at A, if AB = 5, AC = 12 and BC = 13, find sinB, cosC and tanB.
1 1
9. Prove + = 2sec2 .
1 + sin 1 sin
10. Find the mode of the following distribution of marks obtained by 80 students:
Marks obtained 0-10 10-20 20-30 30-40 40-50
Number of students 6 10 12 32 20

SECTION C
1
11. Verify that , 1 and 2 are the zeros of the polynomial 2x3 + x2 5x + 2. Also verify the
2
relationship between the zeros and coefficients.
12. Solve: ax + by = a b; bx ay = a + b
13. The coach of a cricket team buys 7 balls and 6 bats for Rs 3800. Later, he buys 3 balls and 5
bats for Rs 1750. Find the cost of each ball and each bat.
14. State and prove Basic Proportionality Theorem or Thales Theorem.
15. In the given figure, ABC and DBC are on the same base BC and on opposite sides of BC and
area( ABC ) AO
Q is the point of intersection of AD and BC. Prove that = .
area( DBC ) DO
16. A rhombus of side 20 cm has two angles of 60 each. Find the length of the diagonals.
17. If cosec + cot = m and cosec cot = n, prove that mn = 1.
18. Find the mean of the following frequency distribution:
Classes: 0-20 20-40 40-60 60-80 80-100
Frequency: 15 18 21 29 17
19. If the median of the following frequency distribution is 46, find the missing frequencies.
Variable 10-20 20-30 30-40 40-50 50-60 60-70 70-80 Total
Frequency 12 30 ? 65 ? 25 18 229

a2 b2
20. If sin = 2 2 , find the values of other five trigonometric ratios.
a +b

Material downloaded from http://myCBSEguide.com and http://onlineteachers.co.in


Portal for CBSE Notes, Test Papers, Sample Papers, Tips and Tricks
SECTION D

5
21. Obtain all the zeros of the polynomial 3x4 + 6x3 2x2 10x 5, if two of its zeros are and
3

5
.
3
22. In an equilateral triangle ABC, the side BC is trisected at D. Prove that 9AD2 = 7AB2.
23. If cosec sin = l and sec cos = m, prove that l2m2 (l2 + m2 + 3) = 1.
24. Mr. Sharma tells his son, 7 years ago, I was 7 times as old as you were then. Also, 3 years
from now, I shall be 3 times as old as you will be. Represent this situation algebraically and
graphically.
25. Two isosceles triangles have equal vertical angles and their areas are in the ratio 16:25. Find
the ratio of their corresponding heights.
tan A cot A
26. Prove that + = 1 + tan A + cot A = 1 + sec A cos ecA
1 cot A 1 tan A
27. The following table gives the production yield per hectare of wheat of 100 farms of a village.
Production yield in kg/hectare 50-55 55-60 60-65 65-70 70-75 75-80
Number of farms 2 8 12 24 38 16
Change the above distribution to more than type distribution and draw its ogive.
28. Without using trigonometric tables, evaluate the following:
sec37
+ 2cot15 cot 25 cot 45 cot75 cot 65 3(sin2 18 + sin2 72)
cosec53
29. For what value of k will the following system of linear equations has no solution:
3x + y = 1; (2k 1)x + (k 1)y = 2k + 1
30. Prove that in a triangle if the square of one side is equal to the sum of the squares of the other
two sides, then the angle opposite to the first side is a right angle.
31. In a primary school in a village, number of girls enrolled has tripled this year as compared to
last year.
a. Form a linear equation by taking number of enrollment of this year as x and of previous
year as y.
b. If the total number of students enrolled this year is 60, out of which 15 are boys. Find the
number of enrollment of girls of the previous year.
c. Which value is depicted in the question?

Material downloaded from http://myCBSEguide.com and http://onlineteachers.co.in


Portal for CBSE Notes, Test Papers, Sample Papers, Tips and Tricks
CBSE Sample Paper -05 (unsolved)
SUMMATIVE ASSESSMENT I
Class X Mathematics

Time allowed: 3 hours Maximum Marks: 90

General Instructions:

a) All questions are compulsory.


b) The question paper comprises of 31 questions divided into four sections A, B, C and D. You
are to attempt all the four sections.
c) Questions 1 to 4 in section A are one mark questions. These are MCQs. Choose the correct
option.
d) Questions 5 to 10 in section B are two marks questions.
e) Questions 11 to 20 in section C are three marks questions.
f) Questions 21 to 31 in section D are four marks questions.
g) There is no overall choice in the question paper. Use of calculators is not permitted.

SECTION A

1. If n is a natural number, then 92n 42n is always divisible by


a. 5 b. 13 c. both 5 and 13 d. None of these
2. If and are the zeros of the quadratic polynomial f(x) = ax2 + bx + c, then evaluate 2 + 2.
3. The areas of two similar triangles ABC and DEF are 16 cm2 and 25 cm2, respectively. If BC =
2.3 cm, find EF.
4. Write the maximum and minimum values of cos.
5. What is the value of (1 cos2)cosec2?

SECTION B

x cosec2 30 sec2 45
6. If = tan2 60 tan2 30 , then find the value of x.
8cos2 45 sin2 60
7. In a triangle ABC, AC > AB, D is the mid-point of BC and AE BC. Prove that
1 2
AB2 = AD2 BC.DE + BC .
4

Material downloaded from http://myCBSEguide.com and http://onlineteachers.co.in


Portal for CBSE Notes, Test Papers, Sample Papers, Tips and Tricks
8. In the given figure, find tanP and cotR. Is tanP = cotR?

9. In the given figure, ABCD is a rectangle. Find the values of x and y.

10. Find the mode of the following data:


Class 0-20 20-40 40-60 60-80
Frequency 15 6 18 10

SECTION C

11. A library has a fixed charge for the first 3 days and an additional charge for each day
thereafter. Sarika paid Rs 27 for a book kept for 7 days while Suruchi paid Rs 21 for the book
kept for 5 days. Find the fixed charge and the charge for each extra day.

12. AD is an altitude of an equilateral triangle ABC. On AD as base, another equilateral triangle


ADE is constructed. Prove that Area (ADE) : (ABC) = 3 : 4.

13. Prove that 7 is an irrational number.

tan cot
14. Prove that = tan2 cot 2 .
sincos

Material downloaded from http://myCBSEguide.com and http://onlineteachers.co.in


Portal for CBSE Notes, Test Papers, Sample Papers, Tips and Tricks
15. In the given figure, AB BC, FG BC and DE AC. Prove that ADE GCF.

16. The sum of the numerator and denominator of a fraction is 8. If 3 is added to both the
3
numerator and denominator, the fraction becomes . Find the fraction.
4
17. Find the missing frequencies in the following frequency distribution table, if N = 100 and
median is 32.
Marks obtained 0-10 10-20 20-30 30-40 40-50 50-60 Total
Number of students 10 ? 25 30 ? 10 100

18. If and are the zeros of the polynomial x2 2x 15, then form a quadratic polynomial
whose zeroes are 2 and 2.
19. Show that any positive odd integer is of the form 4q + 1 and 4q + 3 where q is a positive
integer.
20. If cos + sin = 2 cos, show that cos sin = 2 sin.

SECTION D

21. Find all the zeros of the polynomial 2x4 + 7x3 19x2 14x + 30, if two zeros are 2 and 2 .
22. Find the value of k so that the following linear equations have no solution:
(3k + 1)x + 3y 2 = 0
(k2 + 1)x + (k 2)y 5 = 0
23. If a line is drawn parallel to one side of a triangle to intersect the other two sides in distinct
points, prove that the other two sides are divided in the same ratio.
24. The path of train A is given by the equation x + 2y 4 = 0 and the path of train B is given by
the equation 2x + 4y 12 = 0. Represent this situation graphically.
25. If 2cos sin = x and cos 3sin = y. Prove that 2x2 + y2 2xy = 5.

Material downloaded from http://myCBSEguide.com and http://onlineteachers.co.in


Portal for CBSE Notes, Test Papers, Sample Papers, Tips and Tricks
26. Use euclids division algorithm to find the HCF of 10224 and 9648.
27. The following table shows the marks scored by 140 students in an examination of a certain
paper:
Marks 0-10 10-20 20-30 30-40 40-50
Number of students 20 24 40 36 20
Find the mean marks by using all the three methods: direct method, assumed mean deviation
and shortcut method.
seccosec ( 90 ) tancot ( 90 ) sin2 55 + sin2 35
28. Evaluate .
tan10 tan20 tan60 tan70 tan80
29. Divide 15x3 20x2 + 13x 12 by 2 2x + x2 and verify the result by division algorithm.
30. In the given figure, each of PA, QB, RC and SD is perpendicular to l. If AB = 6 cm, BC = 9 cm,
CD = 12 cm and PS = 36 cm, then find PQ, QR and RS.

31. A survey was conducted to find out the time spent by ten friends in an orphanage. The
recordings are as given below:
10, 7, 13, 10, 14, 12, 20, 15, 10, 16
a. Find the mean, mode and median.
b. Which value we draw from the above data?

Material downloaded from http://myCBSEguide.com and http://onlineteachers.co.in


Portal for CBSE Notes, Test Papers, Sample Papers, Tips and Tricks

Potrebbero piacerti anche